You are on page 1of 62

USMLE WORLD STEP 1

GENETICS
Question List

Genetics Q No:
Genetics Q No:
Genetics Q No:
Genetics Q No:
Genetics Q No:
Genetics Q No:
Genetics Q No:
Genetics Q No:
Genetics Q No:
Genetics Q No:
Genetics Q No:
Genetics Q No:
Genetics Q No:
Genetics Q No:
Genetics Q No:
Genetics Q No:
Genetics Q No:
Genetics Q No:
Genetics Q No:
Genetics Q No:
Genetics Q No:
Genetics Q No:
Genetics Q No:
Genetics Q No:
Genetics Q No:

1
2
3
4
5
6
7
8
9
10
11
12
13
14
15
16
17
18
19
20
21
22
23
24
25

Pulmonology
Endocrinology
Renal
Reproductive system
Pulmonology
Head and neck
Head and neck
Neurology
Hematology
Endocrinology
Neurology
Head and neck
Cardiology
Renal
Cardiology
Pulmonology
Head and neck
Cardiology
Head and neck
Reproductive system
Oncology
Neurology
Head and neck
Musculoskeletal
Genitourinary

Genetics Q No:
Genetics Q No:
Genetics Q No:
Genetics Q No:
Genetics Q No:
Genetics Q No:
Genetics Q No:
Genetics Q No:
Genetics Q No:
Genetics Q No:
Genetics Q No:
Genetics Q No:
Genetics Q No:
Genetics Q No:
Genetics Q No:
Genetics Q No:
Genetics Q No:
Genetics Q No:
Genetics Q No:
Genetics Q No:
Genetics Q No:
Genetics Q No:
Genetics Q No:
Genetics Q No:
Genetics Q No:

26
27
28
29
30
31
32
33
34
35
36
37
38
39
40
41
42
43
44
45
46
47
48
49
50

Musculoskeletal
Dermatology
Hematology
Gastrointestinal system
Pulmonology
Dermatology
Endocrinology
Neurology
Neurology
Musculoskeletal
Neurology
Neurology
Gastrointestinal system
Neurology
Neurology
Cardiology
Hepatobiliary system
Hematology
Hematology
Reproductive system
Neurology
Neurology
Head and neck
Head and neck
Hematology

161

USMLE WORLD STEP 1

GENETICS

Q NO 1: Scientists develop a technique that promotes intracellular folding and


glycosylation of an abnormal transmembrane protein that would otherwise be
degraded before reaching the cell surface. This technique would be most
helpful in treating patients with:
Phenylketonuria
B. Sickle cell anemia
C. Marfan syndrome
D. Cystic fibrosis
E. Fredericks ataxia

A.

Explanation:
F508 is the most common CETR gene mutation in patients with cystic fibrosis (CE).
This mutation deletes the three nucleotides that code for phenylalanine at amino
acid position 508. F508 is the mutation responsible in approximately 70% of CF
cases diagnosed worldwide.
The CETR gene codes for the cystic fibrosis transmembrane regulator (CETR)
protein, an integral membrane protein. The F508 mutation causes abnormal protein
folding and failure of glycosylation. The CRTR protein is degraded before it reaches
the cell surface, causing its complete absence from the apical membrane of exocrine
ductal epithelial cells. The molecular technique described in this question could help
CF patients by restoring CFTR proteins to the membrane.
(Choice A) Phenylketonuria results from mutations in the hepatocyte intracellular
enzyme phenylalanine hydroxylase. The result is protein misfolding. This enzyme is
not normally transported to the cell surface.
(Choice B) Sickle cell anemia results from mutations in the -globin gene of
hemoglobin S. This protein is not a transmembrane protein.
(Choice C) Marfan syndrome results from an inherited defect in the extracellular
glycoprotein, fibrillin-1. Fibrillin-1 is not an integral transmembrane protein. The
abnormal fibrillin-1 monomers disruptfibrillin-1 polymerization and prevent the
normal formation of microfibrils in interstitial connective tissue.
(Choice E) In Fredericks ataxia, the frataxin gene, which codes for a mitochondrial
protein important in respiratory function and iron homeostasis, is mutated. Here, a
GAA repeat disrupts transcription, not post-translational processing.
Educational Objective:
In most cases of CF, the mutation in the CFTR gene product causes defective posttranslational folding and glycosylation. The result is degradation of the CETR integral
membrane protein before it reaches the cell surface.

162

USMLE WORLD STEP 1

GENETICS

Q NO 2: A 15-year-old patient is referred to your office by a teacher who is


concerned about the patients learning abilities. After evaluating the patient,
you diagnose mild mental retardation. Cytogenetic studies of the patients
buccal mucosal cells reveal a 47 XXY karyotype. Further evaluation is likely
to reveal:
stature, broad chest, amenorrhea
B. Tall stature, gynecomastia, infertility
C. Macroorchidism, large jaw and ears
D. Arachnodactyly, scoliosis, aortic root dilation
E. Short stature, hypotonia, obesity

A.
Short

Explanation:
The 47 XXY karyotype is diagnostic of Klinefelter syndrome. The extra sex
chromosome is acquired due to meiotic nondisjunction during gametogenesis.
Variants include 48 XXXY and 46 XY/46 XXY mosaicism. Neonates are pheno
typically normal. Signs do not become evident until puberty. The major features of
this disorder are described below:
1. Klinefelter syndrome causes primary testicular failure due to hyalinization and
fibrosis of the seminiferous tubules and the subsequent lack of testosterone
synthesis. The testes are small and firm. These abnormalities result in
oligo/azoospermia, infertility, and absence of secondary sex characteristics.
Gonadotropin (FSHI LH) levels are increased, and the testosterone concentration is
low.
2. Testosterone deficiency also leads to the characteristic eunuchoid body habitus.
Patients have tall stature and gynecomastia. Facial and body hair is absent and
muscle mass is decreased.
3. Cognitive symptoms: Mild mental retardation is seen in some patients, though the
majority has normal intelligence. The risk and severity of mental retardation
increases with each additional X chromosome.
(Choice A) Patients with Turners syndrome have short stature, broad chest, and
primary amenorrhea. Only females are affected. Most of the patients have the
karyotype 45 XO, though some are 45 X0/46 XX mosaics.
(Choice C) Macroorchidism, large jaw and mental retardation are seen in patients
with fragile X syndrome, a common cause of hereditary mental retardation, inherited
as an X-linked trait.
(Choice D) Arachnodactyly, scoliosis and aortic root dilation are signs of Marfan
syndrome, which occurs due to an inherited defect of the extracellular matrix protein
fibrillin.
(Choice E) Patients with Prader-Willi syndrome have short stature, hypotonia and
obesity. This syndrome occurs due to a micro deletion on chromosome 15 that is
inherited from the father. Patients are mentally retarded.
Educational Objective:
47 XXY is the most common karyotype producing Klinefelter syndrome. Patients
present with tall stature, small firm testes, azoospermia and gynecomastia. Mild
mental retardation may be present.

163

USMLE WORLD STEP 1

GENETICS

Q NO 3: Bilateral renal masses composed of fat, smooth muscle and blood vessels
are detected in a young Caucasian woman. The patient is most likely to suffer
from:
A. Brain hamartomas and ash-leaf skin patches
B. Multiple peripheral neuro fibromas and caf au lait skin spots
C. Bilateral acoustic neuromas
D. Cerebellar hemangiomas and liver cysts
E. Capillary angiomas of the face and choroid
F. Multiple telangiectasias of the skin and mucosa
Explanation:
Renal angiomyolipoma is a benign tumor composed of blood vessels, smooth
muscle, and fat (angio-myo-lipoma). These tumors can be diagnosed with an
abdominal CT scan, as the density of fat is less than that of water.
Angiomyolipomas are associated with tuberous sclerosis. In patients with bilateral
renal angiomyolipomas, the incidence of tuberous sclerosis is 80-90%. Tuberous
sclerosis is an autosomal dominant condition characterized by cortical tubers and
subependymal hamartomas in the brain, with consequent seizures and mental
retardation. Cardiac rhabdomyomas, facial angiofibromas, and leat-shaped patches
of skin lacking pigment (ash leaf patches) can occur in this condition as well.

(Choice B) Neurofibromatosis Type 1


is an autosomal dominant disorder
characterized by neurofibromas
(plexiform and solitary), optic
gliomas, pigmented nodules of the
iris (Lisch nodules), and cutaneous
hyperpigmented macules (caf au
lait spots).

(Choice C) Neurofibromatosis Type 2 is an autosomal dominant disorder marked by


bilateral acoustic neuromas. These patients may also develop multiple meningiomas,
gliomas, and ependymomas of the spinal cord.
(Choice D) Cerebellar hemangiomas and liver cysts are seen in Von Hippel-Lindau
Disease, an autosomal dominant condition. These patients are at high risk for
bilateral renal cell carcinomas.
(Choice E) Sturge-Weber syndrome (SVVS) is a rare congenital vascular disorder
characterized by a facial port-wine stain and a leptomeningeal angioma.
(Choice F) Patients with hereditary hemorrhagic telangiectasia (also called OslerRendu-Weber syndrome) develop multiple telangiectasias of the skin and mucosa.
The typical presentation is recurrent epistaxis or gastrointestinal bleeding (melena).
Educational Objective:
Renal angiomyolipoma is a benign tumor composed of blood vessels, smooth
muscle, and fat. Bilateral renal angiomyolipomas are associated with tuberous
sclerosis, an autosomal dominant condition.

164

USMLE WORLD STEP 1

GENETICS

Q NO 4: A 17-year-old Caucasian female has never had menstrual bleeding. She is


148 cm tall, weighs 43 kg (94.7 lb), and has poorly developed secondary sexual
characteristics. On pelvic ultrasound, her ovaries are small and elongated. Which of
the following is the most likely cause of this patients condition?
A. Mitotic error in early development
B. Trinucleotide repeat expansion
C. Uni parental disomy
D. Balanced reciprocal translocation
E. Frameshift mutation
Explanation:
This patient appears to have Turner syndrome, a condition that occurs in
approximately 1 in 2000 liveborn females (and in 15% of all spontaneous abortions).
Clinical manifestations of Turner syndrome include the abnormalities listed below.
1. Phenotypic abnormalities: Short stature, webbed neck, high palate, low posterior
hair line, shield chest, widely-spaced nipples, short fourth metacarpal
2. Urogenital abnormalities: prima amenorrhea, underdeveloped streak ovaries,
decreased estrogen and increased gonadotropins, absent secondary sex
characteristics, horseshoe kidney
3. Cardiac abnormalities: coarctation of the aorta, bicuspid aortic valve
4. Lymphatic abnormalities: edema of extremities in neonates, cystic hygromas
Turner syndrome is genetically heterogeneous. The classic variant of complete
monosomy (45, XO) occurs in a minority of Turner syndrome patients, with most
demonstrating mosaicism. In the mosaic population, one genetic line contains cells
with a normal number of chromosomes (46 XX), while the other genetic line contains
cells that are monosomic (45, XO). Both lines originate from a single zygote. While
trisomies and nonmosaic monosomies typically result from meiotic nondisjunction,
mosaicism arises secondary to mitotic errors after fertilization has taken place.
Some individuals with Turner syndrome have normal karyotypes (46 XX) with partial
deletion of one or more arms of the X chromosome. This defect is termed partial
monosomy and is associated with the standard clinical manifestations of Turner
syndrome.
(Choice B) Trinucleotide repeat expansion is responsible for numerous diseases,
including fragile X syndrome (CGG repeats), myotonic dystrophy (CTG repeats), and
Huntington disease (CAG repeats).
(Choice C) Uniparental disomy occurs when an individual inherits two copies of a
chromosome from one parent and no copies of the chromosome from the other
parent. Prader-Willi syndrome occurs in those who have two maternal copies of
chromosome 15 (maternal uniparental disomy) and Angelman syndrome occurs in
those who have two paternal copies of chromosome 15 (paternal uniparental
disomy). Uniparental disomy is not associated with Turner syndrome.
(Choice D) In balanced form, a reciprocal (F robertsonian) translocation is clinically
silent because there is no excess or shortage of genetic material. An unbalanced
trisomy 21 (in which one chromosome 14 contains the long arms of both
chromosomes 14 and 21) is responsible for 3-4% of cases of Down syndrome.
(Choice E) A frameshift mutation is a genetic mutation in which a number of
nucleotides not divisible by three is inserted or removed from the DNA sequence. As
a result, the reading frame is disrupted and the product of translation is changed.
Familial hypercholesterolemia has been associated with frameshift mutations.

165

USMLE WORLD STEP 1

GENETICS

Educational Objective:
Patients with Turner syndrome may have karyotype 45, XO (complete monosomy),
45X0/46XX (mosaicism), or 46XX (with partial deletion of one X chromosome).
Mosaicism appears to account for a majority of cases of Turner syndrome.

166

USMLE WORLD STEP 1

GENETICS

Q NO 5: A young couple presents to your office seeking prenatal counseling. The


husband suffers from cystic fibrosis and is very concerned that his child wills in
her it the disease. The family pedigree is diagrammed below with the unborn
child marked by a red arrow. What is the chance that this child will inherit cystic
fibrosis?
A. 1/16
B. 1/8
C. 1/4
D. 1/3
E. 2/3
F. 3/4
Explanation:
Cystic fibrosis results from an autosomal recessive defect in the cystic fibrosis
transmembrane conductance regulator (CFTR) gene on chromosome 7. From the
pedigree above, we are asked to calculate the probability that a future child will
inherit a mutant allele from each parent. Since the father is homozygous for the
mutant CETR allele, p1 (the probability that the father will transmit the mutant
allele) = 1. Because the mother has an affected sibling we know that she must have
had two heterozygous carrier parents. Thus, the probability, p2, that the mother is a
carrier= 2/3 1as is evident from the diagram below:

If the mother is a carrier, the probability, p3, that she will transmit the mutant allele
to the child is 1/2. Finally the probability that the child will inherit two mutant
alleles, i.e. the probability of inheriting a mutant allele from the father and from the
mother, is p1 x p2 x p3 = 1/3.
Educational Objective:
The probability that an autosomal recessive disease will be transmitted to a child can
be calculated from the maternal and paternal pedigrees.

167

USMLE WORLD STEP 1

GENETICS

Q NO 6: An infant born prematurely to a 38-year-old Caucasian female is small for


gestational age. Physical examination reveals a bilateral cleft lip, microcephaly, and
microphthalmos. Viscera protrude from an umbilical opening in the childs
abdominal wall. Which of the following karyotypes is most likely in this case?
A. Trisomy 21
B. Trisomy 18
C. Trisomy 13
D. 47, XXX
E. 47, XXY
F. 47, XYY
Explanation:
Trisomy 13 (Patau syndrome) is the third most common autosomal trisomy identified
in liveborn infants, and the most severe. Affected children typically die within the
first week of life, with only 5% surviving the first six months. In the majority of
trisomy 13 infants cytogenetic studies demonstrate nondisjunction (47XX, +13); this
chromosomal abnormality arises during maternal meiosis I and is associated with
advanced maternal age.
The prominent phenotypic features of trisomy 13 are associated with an early defect
in prechordal mesoderm development. As a result, the midface, eye, and forebrain
are most markedly affected. The clinical manifestations of Patau syndrome include
the abnormalities categorized by system below.
1. Head and neck: severe cleft lip and/or palate microphthalmia or anophthalmia,
coloboma, cyclops, malformed or absent nose, deafness, scalp defects (aplasia cutis)
2. CNS: severe mental retardation microcephaly, holoprosencephaly (failure of brain
to divide into hemispheres) absent olfactory nerve or bulb, neural tube defects
3. Extremities: polydactyly, rocker-bottom feet
4. Cardiac: PDAI atrial septal defect, ventricular septal defect
5. Renal: polycystic kidney disease
6. Gastrointestinal: abdominal wall defects associated with omphalocele or umbilical
hernia, pyloric stenosis
(Choice A)The fetus with Down syndrome (trisomy 21) will typically have a flat face,
abnormal ears, slanted palpebral fissures, redundant skin at nape of neck,
hypotonia, pelvic dysplasia, and single transverse palmar crease. Duodenal atresia
and tracheoesophageal fistula are two of the more frequently observed
gastrointestinal conditions.
(Choice B) Clinical manifestations of trisomy 18 (Edwards syndrome) include
prominent occiput micrognathia, small mouth low-set and malformed ears, and
rocker-bottom feet. Clenched hands with the index finger overriding the middle
finger and the fifth finger overriding the fourth finger are characteristic for this
condition. Meckels diverticulum and malrotation are common gastrointestinal
abnormalities.
(Choice D) The 47, XXX karyotype is typically clinically silent. Some affected women
may have slightly decreased IC scores. Female newborns with this karyotype are
pheno typically normal with no obvious dysmorphism.
(Choice E) The 47, XXY (Klinefelter syndrome) may be associated with mild mental
retardation or normal intelligence. The typical patient is a tall male adult with
gynecomastia, small testes, and infertility.

168

USMLE WORLD STEP 1

GENETICS

(Choice F)The 47, XYY karyotype is characterized by tall stature, severe acne, and
mild delays in both motor and language development. Male newborns with this
karyotype are pheno typically normal with no obvious dysmorphism.
Educational Objective:
Trisomy 13 (Patau syndrome) most often occurs secondary to nondisjunction during
maternal meiosis I. A severe condition trisomy 13 is strongly associated with cleft lip
and palate, polydactyly, rocker-bottom feet and holoprosencephaly.

169

USMLE WORLD STEP 1

GENETICS

Q NO 7: A female infant born to a 28-year-old woman following an uncomplicated


vaginal delivery is found to have swelling of the hands and a posterior neck mass.
The mass is composed of cystic spaces separated by connective tissue rich in
lymphoid aggregates. The infant most likely suffers from:
A. Cri du chat (5p-) syndrome
B. Downs syndrome
C. Turners syndrome
D. Adrenogenital syndrome
E. Testicular feminization syndrome
Explanation:
The neck mass described in the question is likely a cystic hygroma. Cystic hygromas
are tumors most commonly located on the neck (especially in the posterior triangle)
that are apparent at birth. Cystic hygromas are often seen in infants with Turners
syndrome. Swelling of the hands and feet (lymphedema) is another common early
manifestation of this condition. Both cystic hygromas and lymphedema occur due to
abnormalities of lymphatic outflow in Turners syndrome. The swelling decreases
with age.
Older children and adults with Turners syndrome have a characteristic appearance
that includes: webbed neck, low hairline, and shield chest, short fourth metacarpal
and short stature. Primary amenorrhea is the result of ovarian underdevelopment
(streak ovaries). Patients fail to develop secondary sex characteristics. Aortic
coarctation is a cardiovascular defect commonly found in Turners syndrome.
(Choice A) Patients with cri du chat (5p-) syndrome typically have a round face, catlike cry, and microcephaly.
(Choice B) Neonates with Downs syndrome have a characteristic appearance as well
including a flat face, oblique palpebral fissures, and epicanthal folds. Downs
syndrome is not associated with lymphedema or cystic hygromas.
(Choice D) In adrenogenital syndrome (congenital adrenal hyperplasia), patients
have abnormal sexual differentiation due to defective hormone synthesis in the
adrenal glands. The most common adrenogenital syndrome is 21- hydroxylase
deficiency. Patients present at birth with ambiguous genitalia (females only) and
symptoms of salt wasting.
(Choice E) Testicular feminization syndrome (also called androgen insensitivity
syndrome) occurs due to a defect in testosterone receptors. Neonates with
karyotype 46 XY appear to be pheno typically normal females. There is a blindended vaginal pouch without a uterus.
Educational Objective:
Turners syndrome (45 XO) manifests in the neonate with lymphedema and cystic
hygromas. Short stature, primary amenorrhea and coarctation of the aorta are the
other important clinical features of Turners syndrome in adults.

170

USMLE WORLD STEP 1

GENETICS

Q NO 8: A 42-year-old Caucasian male who was previously diagnosed with


depression is now experiencing involuntary grimacing and strange
movements of his arms and legs. Neurological examination shows normal
strength and normal deep tendon reflexes. No sensory deficits are noted.
The patients grandfather died of a neurological disease at 65 years old,
and the patients father died of a similar disease at 58. That this patient
has an earlier on set of disease than either his father or grandfather is
most likely explained by which of the following?
Increased penetrance
B. Pleiotropy
C. Anticipation
D. Mosaicism
E. Partial deletion
F. Genomic imprinting

A.

Explanation:
Huntington disease manifests with the triad of movement disorder (chorea),
behavioral abnormalities (aggressiveness, apathy or depression), and dementia.
Huntington disease is transmitted as an autosomal dominant trait with 100%
penetrance, meaning that if a child inherits the abnormal gene, that child will
inevitably develop Huntington disease. Most patients develop symptoms in their4os
or5os. An earlier age of onset is associated with a larger number of trinucleotide
repeats. During spermatogenesis, CAG repeats in the abnormal HD gene rapidly
increase. Thus, patients who receive an abnormal gene from their fathers tend to
develop the disease earlier in life. (The number of trinucleotide repeats on HD gene
remains the same during maternal transmission.) The tendency for clinical
symptoms to worsen and/or occur earlier in subsequent generations is called
anticipation.
Anticipation is common in disorders associated with trinucleotide repeats, as in
Fragile X syndrome, myotonic dystrophy, and Friedreich ataxia.
(Choice A) The transmission of an abnormal gene from a parent to a child does not
always cause disease. For example women with BRCA-2 mutations do not always
develop breast cancer. The likelihood that the properties of a gene will be expressed
is called penetrance. Huntington disease is a disorder with 100% penetrance; all
individuals who have an abnormal HD gene will develop Huntington disease.
(Choice B) Sometimes, one gene mutation leads to multiple phenotypic
abnormalities, a genetic phenomenon named pleiotropy. In Huntington disease,
pleiotropy is clearly at work because the mutation of one gene (HD) causes
dysfunction of behavior, of movement, and of cognition.
(Choice D)The presence of two populations of cells with different genotypes in one
patient is called mosaicism. Examples of mosaicism include milder forms of Turner
(genotype 46XX/45X0), Klinefelter (46XY/47XXY), and Down syndromes.
(Choice E) Deletion is the loss of genetic material. Examples of deletion are
DiGeorge syndrome (22q 11 micro deletions) and cri-du-chat syndrome (5p
deletion).
(Choice F) Genomic imprinting is a selective inactivation of the genes of either
maternal or paternal origin, causing a phenomenon sometimes called parent-oforigin gene expression. Genomic imprinting occurs in Prader-Willi and Angelman
syndromes. Both conditions involve deletion of the same gene on the same
chromosome (15), but yew different clinical manifestations occur depending on the
origin of the mutation. If the deletion comes from the father, Prader-Willi syndrome
occursaffected children show insatiable hunger and thirst and emotional lability;

171

USMLE WORLD STEP 1

GENETICS

whereas, deletions inherited from the mother result in Angelman syndromealso


called happy puppet syndrome because children exhibit jerk movements (like a
marionette) and an almost-pathologically happy disposition.
Educational Objective:
An increased number of trinucleotide repeats on the HD gene is associated with
Huntington disease. The larger the number of the repeats, the earlier the onset of
the disease. Trinucleotide expansion occurs during paternal transmission, causing a
genetic phenomenon called anticipation.

172

USMLE WORLD STEP 1

GENETICS

Q NO 9: A 32-year-old male presents to the ER with recent onset of severe fatigue,


exertional dyspnea and fever. Cytogenetic studies of this patients blood cells
demonstrate a 15; 17 chromosomal translocation. Which of the following proteins is
most likely malfunctioning in the affected cells?
A. Epidermal growth factor receptor
B. Platelet derived growth factor receptor
C. Retinoic acid receptor
D. GTP-binding protein
E. Retinoblastoma gene product
Explanation:
Acute myelogenous leukemia (AML) is characterized by failure of immature myeloid
precursors (myeloblasts) to differentiate into mature granulocytes. It is divided into
eight types, M0 through M7. The M3 variant of AMLI acute promyelocytic leukemia,
is associated with the cytogenetic abnormality t(15;17). Here the gene for retinoic
acid receptor alpha from chromosome 17 is transferred to chromosome 15 in a
location adjacent to the PML (promyelocytic leukemia) gene. Fusion of these two
genes produces a chimeric gene product PML/RARa, which codes for an abnormal
retinoic acid receptor. This abnormal fusion gene product inhibits myeloblast
differentiation, producing acute promyelocytic leukemia.
The clinical manifestations of AML, including anemia (fatigue, pallor),
thrombocytopenia (petechiae, hemorrhages) and neutropenia (fever opportunistic
infections) result from marrow replacement by leukemic cells.
(Choice A) Mutations of the genes that code for the epidermal growth factor
receptors are associated with non-small cell lung carcinoma (erbB1), breast cancer
(erbB2 aka HER2/neu), and some ovarian and gastric tumors.
(Choice B) Defective platelet derived growth factor receptor plays a role in the
pathogenesis of chronic myelomonocytic leukemia.
(Choice D) GTP-binding proteins are involved in cellular signal transduction.
(Choice E) An abnormal Rb gene predisposes to development of retinoblastoma and
osteosarcoma.
Educational Objective:
The cytogenetic defect t(15;17) is associated with acute promyelocytic leukemia
(AML type M3). Translocation of the gene for the retinoic acid receptor alpha from
chromosome 17 to chromosome 15 leads to formation of the fusion gene PML/RAR.
This abnormal fusion gene product inhibits differentiation of myeloblasts and triggers
the development of acute promyelocytic leukemia.

173

USMLE WORLD STEP 1

GENETICS

Q NO 10: A 25-year-old male is being evaluated for infertility. On physical


examination, he has bilateral gynecomastia and small firm testes. His
extremities are abnormally long. Further evaluation will most likely show
which of the following?
Serum LH
Serum FSH
Serum testosterone
Sperm count
A.
Decreased
Decreased
Decreased
Decreased
B.
Normal
Normal
Normal
None
C.
Decreased
Normal
Increased
Decreased
D.
Increased
Increased
Decreased
Decreased
E.
Normal
Decreased
Normal
Decreased
Explanation:
The patient described in this vignette has hypogonadism, small firm testes,
elongated limbs, and gynecomastia, Findings highly suggestive of Klinefelter
syndrome. In Klinefelter syndrome, the seminiferous tubules are progressively
destroyed and hyalinized, causing the testes to be small and firm. Serum inhibin
levels are decreased as a result of the damage to the seminiferous tubules. The
interstitial Leydig cells are damaged as well, resulting in low testosterone levels and
erectile dysfunction. Because of the low circulating levels of testosterone and inhibin,
LH and ESH are elevated (loss of feedback inhibition). The hypogonadism causes
epiphyseal fusion to be delayed, hence the elongated limbs. Most patients with
Klinefelter syndrome are infertile.
The typical genotype in patients with Klinefelter syndrome is 47 XXY, with most of
the clinical manifestations resulting from the extra X chromosome. This genotype
can occur when there is non-disjunction of the sex chromosomes during meiotic
division in either parent.
(Choice A) Low LH and ESH along with low testosterone are consistent with central
hypogonadism, which can result from damage to the hypothalamus or pituitary
gland. The congenital absence of GnRH-secreting neurons is called Kallmanns
syndrome.
(Choice B) A patient with a normal hormonal profile but lacking sperm may have an
obstruction somewhere along the path from the testes to the seminal fluid.
(Choice C) Here LH is suppressed, FSH is normal, and testosterone is elevated, in
the setting of a low sperm count. This combination is typical of treatment with
androgens or anabolic steroids with androgenic properties. High androgen levels
suppress LH secretion, thus decreasing endogenous testosterone production. Despite
the high circulating levels of androgen, these patients have a low sperm count. This
is because high local androgen concentrations are necessary for spermatogenesis,
but here, the local seminiferous tubule concentration of testosterone is suboptimal.
Overtime there is some atrophy of the seminiferous tubules causing a decrease in
testicular size.
(Choice E) In this choice LH is normal, FSH is elevated, and testosterone is normal,
with a low sperm count. This is typical of patient with cryptorchidism (or another
condition that damages the seminiferous tubules, such as orchitis). In these
conditions, the seminiferous tubules are damaged and the interstitial Leydig cells are
preserved. Seminiferous tubule damage leads to elevated FSH levels (secondary to
low inhibin levels). LH and testosterone levels are normal because the testosteroneproducing Leydig cells are not affected.
Educational Objective: Klinefelter syndrome is characterized by hypogonadism,
eunuchoid habitus, small firm testes, and genotype 47 XXY.

174

USMLE WORLD STEP 1

GENETICS

Q NO 11: A study conducted on transgenic mouse models of Huntington disease


attempted to reveal the molecular pathogenesis of this disorder. It was found
that the abnormal huntingtin protein coded for by the HD gene decreases
expression of a number of other genes. As a result, some neurotrophic proteins
are not synthesized. Inhibition of gene expression in Huntington disease occurs
via:
A. DNA acetylation
B. Hyper methylation of histones
C. Thymidine dimerization
D. Mutation of homeodomain gene
E. Suppression of mRNA synthesis
Explanation:
Huntington disease is an autosomal-dominant neurodegenerative disease with
complete penetrance that has been localized to a region on the fourth chromosome.
It develops due to an increase in the number of CAG trinucleotide repeats on the HD
gene that codes for the abnormal protein huntingtin. This abnormal gene product
functions to decrease the expression of other genes by inhibiting their transcription.
This is called transcriptional repression or silencing, and plays an important role in
pathogenesis of Huntington disease. Although all cells in the human body have the
same genome, they develop into different variants specific for every organ and
tissue. Such differentiation is possible due to expression of some genes and
silencing of others. This regulation of transcription occurs in part due the presence
of histones, small proteins that complex with DNA to make certain regions of the
genome not accessible for transcription. Methylation of amino acid residues in
histones increases their ability to repress transcription, and in Huntington disease
some histone fragments are hyper methylated leading to increased silencing of
certain genes. Recent research on mouse models has shown that the antimetabolite
mithramycin decreases methylation of histones and may prove useful in the
treatment of Huntington disease.
(Choice A) Acetylation of histones and DNA weakens the DNA-hi stone bond and
make DNA fragments accessible for transcription factors and RNA polymerases.
Acetylation, therefore, enhances gene transcription while methylation decreases
transcription.
(Choice C) Thymidine dimerization is one of the mechanisms of DNA damage
induced by UV radiation.
(Choices D and E) Mutation of homeodomain (homeobox) genes would theoretically
cause alterations in the body structure of an animal or lead to spontaneous abortion
by leading to a body structure incompatible with life. Suppression of mRNA synthesis
would lead to failure of protein synthesis and has not been found to play any role in
Huntington disease.
Educational Objective:
Alteration of gene expression in Huntington disease is believed to occur due to hyper
methylation of histones. Hyper methylated histones bind DNA and prevent its
transcription leading to disruption of synthesis of some neurotrophic proteins.

175

USMLE WORLD STEP 1

GENETICS

Q NO 12: An infant born prematurely to a 42-year-old Caucasian female is small for


gestational age. Physical examination reveals microcephaly, low-set ears, prominent
occiput and small mandible. The infants fists are clenched and the fingers overlap. A
bilateral foot deformity is observed. Which of the following is the most likely
karyotype abnormality in this infant?
A. Trisomy 21
B. Trisomy 18
C. Trisomy 13
D. 47, XXX
E. 47, XXY
Explanation:
Trisomy 18 (Edwards syndrome) is the second most common autosomal trisomy
identified in liveborn infants, with trisomy 21 the most common. Ninety percent of
Edwardss syndrome cases result from nondisjunction, with cytogenetic studies
demonstrating a full trisomy (47, + 18).
The clinical manifestations of Edwardss syndrome include the abnormalities
categorized by system below.
1. Face: micrognathia, microstomia, eye defects (microphthalmia, cataracts
coloboma), low-set and malformed ears, prominent occiput
2. CNS: microcephaly, neural tube defects (eg, meningocele, anencephaly),
holoprosencephaly, Arnold-Chiari malformation, severe mental retardation, delayed
psychomotor development
3. Musculoskeletal: clenched hands with overlapping fingers (index finger overrides
the middle finger and fifth finger overrides the fourth finger), rocker-bottom feet,
short sternum, and hypertonia
4. Cardiac: ventricular septal defect, patent duct arteriosus
5. Gastrointestinal: Meckels diverticulum, malrotation
Trisomy 18 can be detected prenatally with the use of ultrasound. Suggestive
findings include intrauterine growth restriction and polyhydramnios, especially in a
fetus with abnormal hand arrangement.
(Choice A) The fetus with Down syndrome (trisomy 21) will typically have a flat face,
abnormal ears, slanted palpebral fissures, redundant skin at nape of neck,
hypotonia, pelvic dysplasia, and single transverse palmar crease.
(Choice C) Unlike neonates with Edwards syndrome, neonates with trisomy 13
(Patau syndrome) have cleft lip and palate, polydactyly and omphalocele. Patau
syndrome is not associated with low-set ears and overlapping fingers.
(Choice D) The 47, XXX karyotype is typically clinically silent. Some affected women
may have slightly decreased IQ scores. Female newborns with this karyotype are
phenotypically normal with no obvious dysmorphism.
(Choice E) The 47, XXY (Klinefelter syndrome) may be associated with mild mental
retardation or normal intelligence. The typical patient is a tall male adult with
gynecomastia, small testes, and infertility. Male newborns with this karyotype are
phenotypically normal with no obvious dysmorphism. The clinical findings do not
become apparent until adolescence or adulthood.
Educational Objective:
Neonates with Edwardss syndrome (47, XX, +18) have small jaws (micrognathia),
small eyes (microphthalmia), and malformed and low-set ears. Note that rockerbottom feet are seen, as with Patau syndrome (trisomy 13). The presence of
clenched hands with overlapping fingers is considered one of the distinguishing
features of this syndrome.

176

USMLE WORLD STEP 1

GENETICS

Q NO 13: A neonate born to 41-year-old woman in her 39th week of gestation has a
flattened face and epicanthal folds. The childs echocardiography reveals an
endocardial cushion defect. Which of the following most likely occurred prior to
conception?
A. Meiotic non-disjunction
B. Robertsonian translocation
C. Expansion of trinucleotide repeats
D. Formation of fragile site
E. Inactivation of one chromosome
F. Deletion of chromosomal part
Explanation:
The infant described in this clinical vignette has some typical features of Down
syndrome. Characteristic appearance of these patients includes flat facies, epicanthal
folds, oblique palpebral fissures and a single palmar crease. Congenital heart defects
are seen in 50% of patients with Down syndrome, and endocardial cushion defects
are most commonly observed.
The majority of patients with Down syndrome have trisomy 21 that occurs as a
result of chromosomal nondisjunction (failure of chromosomes to separate) during
the first meiotic division of the ovum. The incidence of this abnormality increases
with maternal age.
(Choice B) Robertsonian translocation occurs due to a break near the centromeres of
two chromosomes, with transfer of genetic material between the chromosomes.
Robertsonian translocation accounts for 3-4% of Down syndrome cases, and is
inherited from one of the parents, most commonly the mother, during conception.
Common translocations are t(14:21) and t(21 22).
(Choice C) Disorders associated with expansion of trinucleotide repeats on the
specific gene include Huntington disease, myotonic dystrophy and fragile X
syndrome.
(Choice D) Formation of a fragile site on X chromosome is characteristic for fragile X
syndrome.
(Choice E) Selective inactivation of maternal or paternal alleles is called imprinting.
As a result of this process, the same genetic abnormality can cause different
disorders when inherited from the mother or the father. Prader-Willi and Angelman
syndromes are the classic examples of such disorders.
(Choice F) Many inherited diseases are caused by loss of part of chromosome
(deletion). Cri-du-chat syndrome (5p deletion), DiGeorge syndrome (22q11
microdeletion) and Prader-Willi syndrome (l5q deletion) are some examples.
Educational Objective:
Down syndrome is associated with characteristic physical exam findings such as a
flattened facies, epicanthal folds, oblique palpebral fissures, single palmar crease,
shortened fifth digit, large tongue and others. Congenital heart defects, especially
endocardial cushion defects, are common in children with Downs syndrome. The
majority of cases occur due to maternal meiotic nondisjunction.

177

USMLE WORLD STEP 1

GENETICS

Q NO 14: A 60-year-old male with painless hematuria is found to have a right-sided


renal mass. There is no significant family history. Cytologic evaluation of the mass
shows malignant cells with chromosome Sp deletion. The deletion most likely
involves which of the following genes?
A. RB
B. VHL
C. NE-1
D. WT-1
E. BRCA-1
Explanation:
Given the presentation of hematuria plus renal mass, the patient most likely has
renal cell carcinoma.
The majority of renal cell neoplasms are sporadic. A small number of cases are
hereditary, such as those associated with Von Hippel-Lindau disease. Von HippelLindau disease is an autosomal dominant disorder characterized by cerebellar
hemangioblastomas, clear cell renal carcinomas and pheochromocytomas. Renal cell
carcinoma develops in nearly 40% of these patients, and is a major cause of death.
All patients with Von Hippel-Lindau disease have a deletion of the VHL gene on
chromosome 3p.
Although Von Hippel-Lindau disease is rare, deletions or mutations of the VHL gene
are relatively common. In fact defects of the VHL gene are found in the majority of
patients (40-50%) with sporadic renal cell carcinoma. The VHL gene is a tumor
suppressor gene. Renal cell carcinoma is the only abnormality associated with
deletions or mutations of this gene.
(Choice A) The RB anti-oncogene is located on chromosome 13. Mutations of this
gene lead to the development of retinoblastoma and osteosarcoma.
(Choice C) NE-i is located on chromosome 17. Mutations of this gene cause
neurofibromatosis type 1.
(Choice D) Mutations of the WI-i anti-oncogene are associated with the development
of Wilms tumor. This gene is located on chromosome 11.
(Choice E) Mutations of the BRCA-1 gene put patients at increased risk for breast
and ovarian cancer.
Educational Objective:
Patients with both sporadic and hereditary (associated with Von Hippel-Lindau
disease) renal cell carcinomas are found to have deletions of the VHL gene on
chromosome 3p.

178

USMLE WORLD STEP 1

GENETICS

Q NO 15: A Caucasian newborn with facial dysmorphia and cleft palate is found to
have a deletion involving the long arm of chromosome 22. These findings are most
consistent with:
A. Kartageners syndrome
B. Tuberous sclerosis
C. DiGeorge syndrome
D. Friedreichs ataxia
E. Marfan syndrome
F. Down syndrome
G. Turners syndrome
Explanation:
Deletions involving the long arm of chromosome 22 can result in facial, cardiac and
immunological abnormalities. DiGeorge syndrome is one such manifestation.
DiGeorge syndrome is defined by thymic aplasia and failure of parathyroid
formation, due to defective embryonic development of the third and fourth
pharyngeal pouches. Patients typically present with hypocalcemic tetany and
recurrent viral and fungal infections due to T-cell deficiency. Cardiac defects
associated with this syndrome are Tetralogy of Fallot and interrupted aortic arch.
Chromosome 22q 11.2 deletions are found in 90% of cases of DiGeorge syndrome.
(Choice A) Kartagener syndrome produces immotile cilia due to an autosomal
recessive microtubular defect. Male infertility, recurrent sinusitis, and bronchiectasis
result. There is an association with situs inversus.
(Choice B) Tuberous sclerosis is an autosomal dominant syndrome characterized by
cutaneous angiofibromas (adenoma sebaceum), seizures, and mental retardation.
Pathological lesions include CNS hamartomas and benign neoplasms, renal and other
visceral cysts, a variety of other hamartomas, and cardiac rhabdomyomas.
(Choice D) Friedreichs ataxia is an autosomal recessive spinocerebellar degeneration
with predominantly spinal ataxia. It is not usually associated with facial or palatal
malformations, but can be associated with hypertrophic cardiomyopathy.
(Choice E) Marfan syndrome is an autosomal dominant defect in a connective tissue
glycoprotein that can cause cystic medical necrosis of the aorta. It is not often
associated with a cleft palate, but rather a high arched palate, crowded teeth and a
narrow face.
(Choice F) Down syndrome results from trisomy 21. Patients have flat facies,
prominent epicanthal folds, and occasionally a cleft palate.
(Choice G) Turners syndrome is an XC sex chromosome disorder resulting in short
stature, webbed neck, ovarian failure, and possible coarctation of the aorta.
Educational Objective:
A variety of genetic disorders can result in facial and/or palatal malformations,
including deletions of the long arm of chromosome 22. However deletions involving
the long arm of chromosome 22 are also associated with DiGeorge syndrome
(congenital thymic and parathyroid aplasia, congenital cardiovascular anomalies).

179

USMLE WORLD STEP 1

GENETICS

Q NO 16: In a small city with a stable Caucasian population, the carrier frequency
for cystic fibrosis is 1/30 Caucasian individuals. In a nearby community, the cystic
fibrosis carrier frequency in Asian individuals is 1/100. What is the probability that
a child born to a mother from the Caucasian community and a father from the
Asian community will suffer from the disease?
A. 1/900
B. 1/1000
C. 1/3000
D. 1/6000
E. 1/12000
Explanation:
The mother in this scenario has a 1/30 probability of carrying the mutant CFTR allele
and the father has a 1/100 probability of carrying the mutant CFTR allele. If either
parent is a carrier the probability that the child will in her it the mutant allele from
that parent is 1/2. To inherit the cystic fibrosis disease the child must independently
inherit a mutant allele from both parents. The probability that the mother is a carrier
and that she will transmit the mutant allele is 1/30x 112 and the probability that the
father is a carrier and will transmit the mutant allele is 1/30x 1/2. The probability
that both events will occur producing a child with cystic fibrosis is the product of the
probabilities of these independent
Events = (1/30x1/2) x (1/100 x 1/2) = 1/12000.
Educational Objective:
After participating in this learning exercise you should be able to calculate the
probability that a child of parents from two populations with different mutant allele
carrier frequencies will inherit an autosomal recessive disease.

180

USMLE WORLD STEP 1

GENETICS

Q NO 17: Virus A cannot normally infect human epithelial cells. After exposure to
virus B in non-human cells, virus A acquires the ability to infect human epithelial
cells; however, the viral particles generated as a result of this infection still cannot
infect human epithelial cells. The phenomenon described is best categorized as
which of the following?
A. Reassortment
B. Recombination
C. Transformation
D. Phenotypic mixing
E. Interference

Explanation:
The acquisition of a new viral surface protein is often all that is necessary for a virus
to infect a new type of host cell. In the illustration above, the virus A genome
obtains some of the surface components of virus B (and vice versa) while both
viruses are present simultaneously in the same human cell. This exchange is defined
as phenotypic mixing, which generally occurs when a host cell is co-infected by two
viral strains and progeny virions are produced that contain nucleocapsid or envelope
proteins from the first strain and the genome of the second strain (or vice versa).
Since virus A acquired some virus B surface proteins but no genetic exchange
occurred, the progeny are considered phenotypically mixed. The next generation will
revert to having only virus A type surface proteins (virus A phenotype) and will again
be non-infectious for human epithelium.
(Choice A) Reassortment refers to changes in genomic composition that occur when
host cells are co-infected with two segmented viruses that exchange whole genome
segments. This process can cause sudden alterations in the surface antigens of the
viral progeny, as observed with the highly mutagenic influenza virus. When
reassortment takes place in a cell co-infected by two viral strains any genomic
change in the first generation progeny that is responsible for new infectivity to
human epithelial cells will also be present in the second generation.
(Choice B) Recombination may be defined as the exchange of genes between two
chromosomes (double-stranded DNA molecules) by crossing over within homologous
regions. When recombination takes place in a cell co-infected by two viral strains any
genomic change in the first generation progeny that is responsible for new infectivity
to human epithelial cells will also be present in the second generation.

181

USMLE WORLD STEP 1

GENETICS

(Choice C) Transformation is generally defined as the uptake of naked DNA by a


prokaryotic or eukaryotic cell. In virology this term may also be used to describe the
incorporation of viral DNA into a host cell chromosome. Transformation alters the
genetic composition of the host cell but typically causes no genomic change in
progeny virions.
(Choice E) Interference is the inhibition by one virus of the replication and/or release
of a second virus that is infecting the same cell. Simple interference would not result
in a new progeny phenotype.
Educational Objective:
Phenotypic mixing refers to co-infection of a host cell by two viral strains resulting in
progeny virions that contain nucleocapsid proteins from one strain and the genome
of the other strain. Since there is no change in the underlying viral genomes (no
genetic exchange) the next generation of virions reverts to their original, unmixed
phenotypes.

182

USMLE WORLD STEP 1

GENETICS

Q NO 18: A 20-year-old woman undergoes a bilateral oophorectomy. The ovaries


are small and underdeveloped. On light microscopic examination, the ovaries are
found to consist of connective tissue with no follicles present. This patient is at
increased risk for which of the following:
A. Mitral valve prolapse
B. Aortic coarctation
C. Lens displacement
D. Vaginal adenosis
E. Pancreatic insufficiency
Explanation:
The ovaries described in the vignette are the streak ovaries of Turners syndrome
(karyotype 45 XO, lacking the paternal X chromosome). In Turners syndrome, the
ovaries develop normally during fetal life. However, lack of the paternal X
chromosome causes loss of follicles by age 2. At puberty, these patients develop
normal pubic hair but fail to undergo breast development or menarche due to
ovarian failure. Turners syndrome is one of the most common causes of primary
amenorrhea. Elevated levels of LH and ESH are characteristic.
Turners syndrome is associated with a number of extragonadal abnormalities,
including:
1. Characteristic appearance: short stature, shield chest (broad, with widely spaced
nipples), webbed neck (broad neck with low hairline)
2. Lymphedema may present at any age. Lymphedema in the neonate is suggestive
of Turners syndrome.
3. Cardiovascular malformations, including coarctation of the aorta, bicuspid aortic
valve, and aortic dissection in adulthood.
(Choices A and C) Mitral valve prolapse and lens displacement are seen in patients
with Marfan syndrome. These patients are also at risk for aortic root dilation and
aortic dissection.
(Choice D) Vaginal adenosis is replacement of the vaginal squamous epithelium with
glandular columnar epithelium. It occurs in the female children of women exposed to
diethylstilbestrol (DES) during pregnancy. Vaginal adenosis is a precursor of clear
cell adenocarcinoma of the vagina.
(Choice E) Pancreatic insufficiency occurs in patients with cystic fibrosis.
Educational Objective:
Streak ovaries amenorrhea and infertility are the gonadal manifestations of
Turners syndrome. This condition is associated with short stature webbed neck, low
posterior hairline and coarctation of the aorta.

183

USMLE WORLD STEP 1

GENETICS

Q NO 19: A stillborn fetus delivered at the 28th week of gestation has flat facial
features and excessive skin at the posterior neck. Autopsy findings include a
ventricular septal defect and duodenal atresia. Which of the following is the most
likely karyotype abnormality in this fetus?
Trisomy 21
B. Trisomy 18
C. Trisomy 13
D. 47, XXX
E. 47, XXY
F. 47, XYY
G. 45, XO

A.

Explanation:
Flat facial features and excessive skin at the nape of the neck are two of numerous
phenotypic features associated with Down syndrome (trisomy2l). Other classic
phenotypic findings include slanted palpebral fissures and a single transverse palmar
crease. Visceral anomalies are common. Cardiac defects are found in approximately
half of all infants with Down syndrome, with the endocardial cushion defect
(atrioventricular septal defect) and ventricular septal defect most often seen.
Gastrointestinal tract abnormalities are also identified in 10-15% of this patient
population, and can include duodenal atresia, Hirschsprungs disease, and
tracheoesophageal fistula.
Down syndrome incidence increases with maternal age. The vast majority of cases
arise due to chromosomal nondisjunction during maternal meiosis I, which results in
a full trisomy 21 present at conception.
(Choice B)Trisomy 18 (Edwards syndrome) often results in fetal death. Clinical
manifestations in liveborn infants include prominent occiput micrognathia, small
mouth, low-set and malformed ears, and rocker-bottom feet. Clenched hands with
the index finger overriding the middle finger and the fifth finger overriding the fourth
finger are characteristic for this condition.
(Choice C)Trisomy 13 (Patau syndrome) often results in fetal death. Clinical
manifestations in liveborn infants include cleft lip and palate polydactyly,
microcephaly, rocker-bottom feet and umbilical hernia. Cardiac and renal defects are
usually present.
(Choices D, E, and F) Karyotypes 47, XXX, 47, XXY (Klinefelter syndrome), and 47,
XYY do not cause death in utero. Newborns with these karyotypes are phenotypically
normal with no obvious dysmorphism.
(Choice G) Stillborn fetuses with Turner syndrome (45XO) are likely to have
edematous hands and feet cystic hygroma of the neck and coarctation of the aorta.
Turner syndrome is not associated with flat facial features ventricular septal defects
or duodenal atresia
Educational Objective:
Down syndrome (trisomy 21) occurs in approximately 1 in 730 live births. The
majority of fetuses with this chromosomal defect die in utero. The triple marker test,
quadruple marker test, and integrated test allow for Down syndrome screening.
Amniocentesis and chromosomal analysis of fetal cells can be used to verify the
diagnosis.

184

USMLE WORLD STEP 1

GENETICS

Q NO 20: A 40-year-old pregnant female at 28 weeks gestation presents with


vaginal bleeding. There are no fetal heart tones audible on Doppler
ultrasound. The tissues evacuated from the patients uterus consist of vesicles
and have a 46 XX genotype. Tissue analysis would most likely reveal
chromosomes derived:
From the father only
B. Half from the father and half from the mother
C. Mostly from the father
D. Mostly from the mother
E. From the mother only

A.

Explanation:
This patient has a hydatidiform mole. A mole is a form of gestational trophoblastic
disease that can be classified as complete or partial. A complete mole has no fetal
structures and is composed entirely of large, edematous and disordered chorionic
villi that appear grossly as clusters of vesicular structures with a bunch of grapes
appearance. Complete moles almost always have a 46 XX karyotype, and all
chromosomes are paternally derived (Choice A). A complete mole forms when a
sperm fertilizes an egg and replicates its own chromosomes while eliminating the
maternal chromosomes. Androgenesis is the term for this process of formation and
survival of tissue of completely paternal derivation.
Unlike in a complete mole, a partial hydatidiform mole shows formation of some fetal
structures grossly. Additionally, a partial mole typically results from fertilization of an
ovum (one set of haploid maternal chromosomes) by two or more sperm (two sets
of haploid paternal chromosomes) resulting in karyotypes like 69 XXY or 69 XXX.
(Choice B) In most normal pregnancies, half of the chromosomes are from the father
and half are from the mother.
(Choices C and D) Conditions caused when a disproportionate amount of
chromosomes are either paternal or maternal include: partial molar pregnancy,
diseases caused by chromosomal nondisjunction such as Down, Edwards, and Patau
syndromes, and Klinefelter (XXY) and XYY syndromes.
Educational Objective:
A complete mole results from fertilization of an ovum that is devoid of genetic
material and subsequent reduplication of the paternal genetic complement giving a
characteristic 46 XX genotype.

185

USMLE WORLD STEP 1

GENETICS

Q NO 21: A 24-year-old recent Russian immigrant describes a long history of weight


loss, night sweats and nagging cough. Imaging and biopsy of his lungs reveal
numerous apical granulomas with central caseous necrosis. Surrounding the necrotic
areas are large cells with abundant pale cytoplasm: these cells most likely have
which of the following surface markers?
A. CD4
B. CD7
C. CD8
D. CD14
E. CD20
Explanation:
This patients history of night sweats, weight loss, and cough together with the
finding of apical pulmonary granulomas showing caseous necrosis are highly
suggestive of secondary tuberculosis. The caseating granulomas of tuberculosis
consist of large epithelioid macrophages with pale pink granular cytoplasm
surrounding a central region of necrotic debris. CD4 is a surface marker of the
monocyte-macrophage cell Image.
(Choice A) Together with T-cell receptors CD4 transmembrane proteins recognize
antigens presented by MHC Class II molecules. CD4 is thus found on the surface of
MHC Il-restricted T-helper cells. Although granulomas may be surrounded by a collar
of mononuclear leukocytes, mainly lymphocytes (possibly including CD4+ T-cells)
and some plasma cells, these are distinct from the large epithelioid macrophages
that are almost always present in granulomas.
(Choice B) CD7 is a multi-chain complex T-cell marker.
(Choice C) CD8 is a transmembrane protein associated with T-cell receptors.
Together these proteins recognize antigenic peptides presented by MHC Class I
molecules. Thus CD8 is found on the surface of MHC I restricted cytotoxic T-cells.
(Choice E) CD20 is a B-cell surface marker. Monoclonal antibodies against the CD-20
antigen (Rituximab) have been successful in the treatment of lymphomas.
Educational Objective:
The caseating granulomas of tuberculosis almost always contain large epithelioid
macrophages with pale pink granular cytoplasm and surface CD14 at the periphery.
CD14 is a surface marker of the monocyte-macrophage cell Image. CD4 and CD8
are surface markers of T-helper and T-killer cells respectively.

186

USMLE WORLD STEP 1

GENETICS

Q NO 22: A Caucasian male presents to your office with mild headaches of recentonset and left leg weakness. The lesions on physical exam are shown in the photo
below.

If inherited, this patients disorder is most likely to be which of the following?


A. Single gene X-linked
B. Single gene autosomal recessive
C. Single gene autosomal dominant
D. Mitochondrial
E. Continuous gene deletion
F. Multifactorial
Explanation:
The multiple small cutaneous lesions shown in the image are most likely
neurofibromas associated with NE type 1. It is an autosomal dominant condition with
high penetrance (most patients who inherit the gene develop the disease). Mutation
of the NF-1 gene, located on chromosome 17, is responsible for this disorder.
The presentation of NF-1 is highly variable: all or none of the following symptoms
maybe present in any individual who suffers from NF-1.
1. Skin: Caf-au-Iait spots are hyperpigmented lesions with either smooth or
irregular borders. The presence of axillary or inguinal freckles is another cutaneous
feature of NE-i.
2. Neurofibromas: sort, sessile or pedunculated lesions that vary in size. They are
commonly multiple and distributed throughout the body.
3. Eye: Optic nerve gliomas may occur and cause visual loss. Lisch nodules are
pigmented hamartomas of the iris and are asymptomatic.
4. Bony abnormalities include sphenoid dysplasia, congenital pseudoarthrosis, and
scoliosis.
5. Other associated tumors: meningiomas, gliomas, pheochromocytomas. This
patients headaches and left leg weakness are probably the result of a meningioma.
Educational Objective:
Neurofibromatosis type l is a single-gene autosomal-dominant disorder. It occurs due
to mutation of the NF-1 gene located on the chromosome i 7. Cafe-au-lait spots,
multiple neurofibromas, and Lisch nodules are the most common symptoms.

187

USMLE WORLD STEP 1

GENETICS

Q NO 23: A stillborn fetus delivered at the 23rd week of gestation is found to have
an edematous neck and broad chest. Autopsy reveals aortal coarctation, a bicuspid
aortic valve, and kidneys that are fused at the midline. Which of the following is the
most likely karyotype abnormality in this fetus?
A. Trisomy 21
B. Trisomy 18
C. Trisomy 13
D. 47, XXX
E. 47, XXY
F. 45, XO
Explanation:
Turner syndrome (45, XO) is the likely diagnosis in a stillborn fetus with nuchal
edema broad chest, and coarctation of the aorta. Most fetuses possessing the 45 XO
karyotype abort spontaneously, with studies suggesting that 15% of all spontaneous
abortions are due to Turner syndrome. The most prominent finding in the neonate or
fetus with Turner syndrome is lymphedema of the hands and feet. Abnormal nuchal
lymphogenesis causes subcutaneous nuchal edema and cystic hygroma in utero. The
lymphedema is also responsible for other common manifestations of Turner
syndrome, including the webbed neck and low posterior hair line. Cardiac
abnormalities associated with Turner syndrome include coarctation of the aorta and
a bicuspid aortic valve. Horseshoe kidney (fusion of kidneys at the midline) is also
identified in many Turner syndrome females. In the adult, classic clinical
manifestations of Turner syndrome include short stature, broad shield-like chest,
webbed neck, and primary ovarian failure. The ovaries become infiltrated with
fibrous tissue (streak ovaries) and fail to produce estrogens.
(Choice A) The fetus with Down syndrome (trisomy 21) will typically have a flat face,
abnormal ears, slanted palpebral fissures, redundant skin at the nape of the neck,
hypotonia, pelvic dysplasia, and single transverse palmar crease.
(Choice B) Trisomy 18 (Edwards syndrome) is the second most common autosomal
trisomy identified in liveborn infants. Clinical manifestations include prominent
occiput, micrognathia, small mouth, low-set and malformed ears, and rocker-bottom
feet. Clenched hands with the index finger overriding the middle finger and the fifth
finger overriding the fourth finger are characteristic for this condition.
(Choice C) Trisomy 13 (Patau syndrome) is the third most common autosomal
trisomy identified in liveborn infants, and the most severe. Affected fetuses typically
have holoprosencephaly, microcephaly, polydactyly and rocker-bottom feet. Multiple
facial abnormalities (eg, hypotelorism, microphthalmia, cleft lip, cleft palate, absent
or malformed nose) frequently occur.
(Choice D) The 47 XXX karyotype is typically clinically silent. Some affected
individuals may have slightly decreased 10 scores. An increased risk of stillbirth is
not associated with this karyotype.
(Choice E) The 47, XXY (Klinefelter syndrome) may be associated with mild mental
retardation or normal intelligence. The typical patient is a tall male with
gynecomastia, small testes, and infertility. An increased risk of stillbirth is not
associated with this karyotype.
Educational Objective:
Lymphedema is a characteristic finding in a fetus afflicted with Turner syndrome (45,
XO). The lymphedema can vary in severity, ranging from edema of the hands and
feet to hydrops fetalis. Other common fetal Turner syndrome abnormalities include
coarctation of aorta and horseshoe kidney.

188

USMLE WORLD STEP 1

GENETICS

Q NO 24: A 5-year-old girl is brought to the physician by her mother. The mother
tells you that the girls skin becomes red and scaling with only minimal sun
exposure. She began to notice this phenomenon when the child was month old.
Now the girls skin is thin and hyperpigmented. The patient has a few nevi on her
hands that have been rapidly enlarging. The defective gene in this patient is
responsible for
A. Regulation of cell cycle
B. Signal transduction
C. DNA excision repair
D. DNA mismatch repair
E. Prevention of micro deletion
F. Regulation of apoptosis
Explanation:
This clinical vignette describes the typical presentation of xeroderma pigmentosum,
which literally means pigmented dry skin. This autosomal recessive (AR) condition
occurs due to decreased ability to repair DNA following damage by UV light. The skin
of affected individuals is normal at birth. The disease manifests during the first year
of life with erythema, scaling and subsequent hyperpigmentation and lentigo
formation on light-exposed areas (especially the face). Later, the skin of affected
areas shows atrophy telangiectasias and intermingling areas of hypo-and
hyperpigmentations. Skin malignancies, including squamous cell carcinoma, basal
cell carcinoma and malignant melanoma, develop as early as at 5-6 years of life.
Normally the regions of DNA damaged by UV radiation are excised and replaced. In
xeroderma pigmentosum, the genes that code for various DNA repair enzymes are
abnormal. Impairment of DNA repair results due to defects in excision of abnormal
nucleotides or defects in replacement of nucleotides following excision.
Other diseases associated with impaired DNA repair include Fanconi anemia (AR,
hypersensitivity to DNA cross- linking agents) and Bloom syndrome (AR,
hypersensitivity to UV damage and chemotherapeutic agents) among others.
(Choice A) The Rb (retinoblastoma) gene is responsible for regulation of the cell
cycle. The abnormal Rb protein loses its ability to arrest the cell cycle in the 01
phase. Mutation of Rb is associated with retinoblastoma and osteosarcoma.
(Choice B) Mutation of ras protein is found in many types of cancers. Ras codes for a
0-protein that regulates signal transduction. Abnormal ras stimulates signal
transduction leading to unregulated cell division, inhibited apoptosis and decreased
cell adhesion. These predispose to malignancy and metastasis.
(Choice D) Abnormalities of genes responsible for DNA mismatch repair are found in
patients with hereditary nonpolyposis colorectal cancer (HNPCC or Lynch syndrome).
These patients have higher incidence of colorectal carcinoma and other
malignancies.
(Choices E and F) Xeroderma pigmentosum is not associated with mutation of genes
that prevent micro deletions or regulate apoptosis though dysregulation of apoptosis
has been implicated in many forms of human cancer.
Educational Objective:
Xeroderma pigmentosum develops due to a defect in DNA excisional repair. This
disease is characterized by increased sensitivity to UV radiation and a high incidence
of all forms of cutaneous malignancy.

189

USMLE WORLD STEP 1

GENETICS

Q NO 25: Triple test performed on a pregnant woman atl8weeks of gestation reveals


low levels of alpha-fetoprotein (AFP). Amniocentesis confirms these findings. The
mother is a known alcoholic and smoker. Low AFP levels are associated with which of
the following conditions?
A. Turner syndrome
B. Omphalocele
C. Trisomy 21
D. Fetal alcohol syndrome
E. Neural tube defects
Explanation:
Down syndrome is the most common chromosomal abnormality, and the second
most common cause of mental retardation. It is associated with advanced maternal
age and can be diagnosed prenatally. Triple test performed at weeks 16-18 of
gestation detects low alpha-fetoprotein (AEP) levels. A finding of low AEP on triple
test is associated with a diagnosis of Down syndrome and is therefore an indication
for amniocentesis. Karyotyping of fetal cells contained in amniotic fluid can diagnose
Down syndrome.
About 90% of patients with Down syndrome have trisomy 21. The parents of these
patients are normal and with the abnormality arising as a consequence of meiotic
non-dysjunction (failure of chromosome 21 to divide during meiosis). This
abnormality occurs in the ovum thus explaining the association of Down syndrome
with increased maternal age.
(Choice B) Omphalocele causes increased APP levels in maternal serum and amniotic
fluid. High APP is also associated with gastroschisis, multiple gestation and neural
tube defects.
(Choice E) Neural tube defects are associated with increased AEP levels and
increased acetylcholinesterase levels in amniotic fluid.
(Choices A and D) Turners syndrome and fetal alcohol syndrome do not cause
abnormal AEP levels.
Educational Objective:
1. Elevated alpha-fetoprotein levels are seen in multiple gestation neural tube
defects (including spina bifida anencephaly), and abdominal wall defects.
2. In contrast Down syndrome is associated with low alpha-fetoprotein levels in
maternal serum and amniotic fluid. The definitive prenatal diagnosis is made by
karyotyping of fetal cells.

190

USMLE WORLD STEP 1

GENETICS

Q NO 26: A married couple presents to your office for routine prenatal counseling.
You notice that the husband is 120 cm (47) tall with disproportionately-short
upper and lower extremities. He cannot provide family history because he was
adopted. His spouse has normal constitutional features and her family history is
insignificant. They are concerned about their unborn childs height. Which of the
following is the best response to their concerns?
A. The risk for the child to be short is about 50%.
B. The risk depends on the mothers carrier status.
C. The risk depends on the childs gender.
D. The risk for the child to be short is about 25%.
E. The condition is not inheritable.
Explanation:
The husband in this vignette has disproportionately-short arms and legs, which
strongly suggests achondroplasia. The most common defect in achondroplasia is an
activating mutation offibrobIastgrothfactorreceptor-3. Patients with achondroplasia
have a normal spine length, but have short limbs, a large head, and sometimes a
saddle nose. Transmission of achondroplasia is autosomal dominant. If one parent
has an abnormal autosomal gene, there is a 50% chance that the parent will
transmit the gene to the child, which means a 50% risk that the child will have the
disease. Sometimes an inherited autosomal dominant trait does not result in the
disease phenotype; this phenomenon is known as incomplete penetration.
(Choices B and C) In sex-linked disorders, the responsible gene is located on a sex
chromosome (either X or Y). Most sex-linked disorders are X-linked recessive. In Xlinked recessive disorders, women with the defective gene will not have the disorder,
but will be carriers. Women with the affected X-linked, recessive gene has a 50%
chance of passing that gene on to their male or female children. The difference is
that male children only have one X chromosome and will be afflicted with the
disorder. Affected female children will be disease-free but will be carriers. The
chances bra carrier female to give birth to an diseased female is virtually 0%; the
chances of her giving birth to an affected male child is about 50%. Males carrying an
X-linked, recessive gene are affected; their daughters always become carriers but
their sons are never affected (because they do not receive an X chromosome from
the father).
In X-linked dominant disorders both males and females are affected. Of the sons and
daughters of an affected female approximately 50% will have the disorder. For an
affected male all of his daughters will be affected and none of his sons will carry or
manifest the disease.

191

USMLE WORLD STEP 1

GENETICS

(Choice D) About 25% of children are affected in autosomal recessive disorders, if


both parents carry the defective gene. A history of consanguinity is sometimes
present in autosomal-recessive disorders. Many of the autosomal recessive disorders
are caused by enzymatic deficiencies. In heterozygotes, the enzyme activity is
reduced by only about 50% an amount of enzymatic activity that is usually sufficient
to prevent the occurrence of disease.
Educational Objective:
Achondroplasia is an autosomal-dominant disorder. The transmission of autosomal
disorders statistically occurs in 50% of the offspring of an affected parent.

192

USMLE WORLD STEP 1

GENETICS

Q NO 27: A young couple who recently immigrated from Eastern Europe brings their
3-year-old son to your office for evaluation of an eczematous rash. On exam you
observe that the child shows signs of mental retardation and has a mousy odor.
What is the likelihood that this couples next child will be affected with the same
disease?
A. Same as the general population
B. 1/32
C. 1/16
D. 118
E. 1/4
F. 1/2
Explanation:
Mental retardation, eczema, and a mousy, musty body odor are signs of
phenylketonuria (PKU). PKU is an autosomal recessive disease caused by mutation
of the gene that code for phenylalanine hydroxylase. In the United States,
phenylalanine levels are measured in all neonates to screen for PKU.
Because PKU is inherited in an autosomal recessive fashion, we know that both of
the parents described above must be heterozygous carriers of the mutation. The
probability that their next child will inherit the disease is thus:
p1 = probability that the mother transmits the mutant allele = 1/2
p2 = probability that the father transmits the mutant allele = 1/2
Because these are independent events, the probability that a child will inherit a
mutant allele from each carrier parent is equal to p1l x p2=%.
Educational Objective:
Mental retardation, eczema, and a mousy or musty body odor in a toddler are
signs of phenylketonuria (PKU). Most infants with PKU are born to two heterozygous
carrier parents. The probability that heterozygous carrier parents will transmit an
autosomal recessive disease like PRU to a child is 74.

193

USMLE WORLD STEP 1

GENETICS

Q NO 28: A 43-year-old male is being evaluated for persistent fever, recurrent gum
bleeding and fatigue. Findings of a peripheral blood smear are shown on the slide
below. Which of the following chromosomal abnormalities is most likely present in
the affected cells?
A. t(8;14)
B. t(9;22)
C. t(15;17)
D. t(11;14)
E. 13q-

Explanation:
This peripheral blood smear shows several myeloblasts, which are large cells with
abundant basophilic cytoplasm. Myeloblasts have a large number of coarse rodshaped intracytoplasmic granules, called Auer rods, a finding characteristic of acute
myelogenous leukemia (AML).
AML is divided into eight types:

The M3 variant of AMLI acute promyelocytic leukemia, is associated with the


cytogenetic abnormality (15; 17). This cytogenetic change causes transfer of the
gene for retinoic acid receptor alpha from chromosome 1]to chromosome 15 in a
location adjacent to the PML (promyelocytic leukemia) gene. Fusion of these two
genes produces a chimeric gene product PML/RARa, which codes for an abnormal
retinoic acid receptor. This abnormal fusion gene product inhibits differentiation of
myeloblasts and triggers the development of acute promyelocytic leukemia.
(Choice A) t(8;14) is associated with Burkitt lymphoma. Here the c-myc
protooncogene is moved from chromosome 8 to chromosome 14, where itis located
adjacent to the gene for immunoglobulin heavy chain. This placement increases the

194

USMLE WORLD STEP 1

GENETICS

production of the oncogene because of the frequency with which the immunoglobulin
gene is transcribed.
(Choice B) Translocation of the abl gene from chromosome 9 to chromosome 22 is
characteristic for chronic myelogenous leukemia (CML). t(9;22) forms the
Philadelphia chromosome and results in the formation of a new gene, bcr-abl,
whose product has tyrosine kinase activity.
(Choice D) Mantle cell lymphoma is a B-cell malignancy associated with t(11;14).
This translocation results in activation of the cyclin D gene.
(Choice E) Deletion of 13q is one of the molecular defects seen in chronic
lymphocytic leukemia (CLL).
Educational Objective:
The presence of rod-shaped intracytoplasmic inclusions known as Auer rods is
characteristic of many forms of acute myeloblastic leukemia (AML). The MS variant
of AMLI acute promyelocytic leukemia, is associated with the cytogenetic
abnormality (15:17).

195

USMLE WORLD STEP 1

GENETICS

Q NO 29: A 23-year-old male with a long history of diarrhea and crampy abdominal
pain is suspected to have ulcerative colitis. A colon biopsy reveals extensive
inflammatory cell infiltration of the mucosa. In the two weeks following the biopsy,
the patients condition improves, despite not receiving any new treatments. A repeat
biopsy reveals markedly decreased inflammation. Which of the following cytokines
most likely contributed to this change?
A. TNF-
B. IL-1
C. IL-4
D. IL-5
E. IL-10
F. IL-12
Explanation:
Essentially, this question asks us to identify which of the listed cytokines has antiinflammatory effects. TGF- and members of the IL-10 family are currently classified
as anti-inflammatory cytokines. Almost all cells express TGF- receptors. TGE-
inhibits Th2 lymphocyte differentiation, cytotoxic T-cell activities, and B-cell
immunoglobulin secretion. It may also suppress NK cells LAK cells, and the
cytotoxicity of mononuclear phagocytes.
In humans, monocytes and B-cells are the major producers of interleukin-10 (IL-10).
IL-10 inhibits IL-2 and IFN-y production by Th1 lymphocytes, IL-4 and IL-5
production by Th2 lymphocytes, and TNF- and IL-12 production by monocytes. It
also decreases NK cell IFN-y production. Moreover, IL-10 inhibits monocyte MHC
class II and B7 expression, impairing the ability of these antigen presenting cells to
activate Th cells. (B7 provides the necessary costimulatory signal.) In the presence
of IL-10, Th lymphocytes may be rendered permanently tolerant to specific antigens.
Educational Objective:
Of the cytokines released in the setting of tissue injury, TGE- and IL-10 are thought
to down regulate local cytokine production and inflammatory reactions contributing
to the systemic acute phase response. IL-1, IL-4, IL-5, and IL-12 stimulate local
immune reactions and may therefore be considered pro-inflammatory. IL-1 also acts
systemically to promote fever and other aspects of the acute phase response.

196

USMLE WORLD STEP 1

GENETICS

Q NO 30: A 4-year-old Caucasian male with recurrent lower lobe pulmonal infiltrates
is found to have high chloride content in his sweat. Which of the following
abnormalities is most likely in this patient?
A. Impaired synthesis of a transmembrane protein
B. Abnormal post-translational processing of a transmembrane protein
C. Decreased epithelial expression of a normal transmembrane protein
D. Poor AIP binding by a transmembrane protein
E. Decreased ion conduction by a transmembrane protein
Explanation:
High sweat chloride concentration is found in cystic fibrosis (CE). CF is characterized
by pancreatic insufficiency, sinopulmonary infections, and malabsorption. The most
common CETR gene abnormality (found in approximately 70% of cases) is a 3-base
pair deletion that removes a phenylalanine at amino acid position 508 (F 508). The
phenylalanine deletion causes abnormal folding and failure of glycosylation. The
abnormal protein is degraded before it reaches the cell surface. The F508 and most
other CETR mutations cause a complete absence of CETR from the apical membrane
of exocrine ductal epithelial cells.
Educational Objective:
Sweat chloride concentrations over 60 mmol/L are found in patients with cystic
fibrosis (CH) F5O8 is the most common CFTR mutation found in patients with CR
This mutation impairs post-translational processing of the CRTR gene transcript. The
result is degradation of the gene product before it can be transported to the cell
surface, causing complete absence of CETR protein from the apical membrane of
exocrine duct epithelial cells.

197

USMLE WORLD STEP 1

GENETICS

Q NO 31: A woman presents to your office complaining of hair loss. She is worried
that she will go bald because both her father and paternal grandmother suffered
from baldness. The most likely inheritance pattern of this condition is:
A. Autosomal dominant
B. Autosomal recessive
C. X-linked
D. Mitochondrial
E. Sporadic

Explanation:
This patient describes a condition with the following inheritance pattern:
The red arrow identifies the case (patient) under consideration. This pedigree could
represent autosomal dominant or X-Linked dominant transmission because the
disease is transferred to both male and female offspring from both male and female
parents, and all generations are affected.
The most common type of hair loss in males and females is androgenetic alopecia
(male pattern baldness). Androgenetic alopecia has autosomal dominant inheritance
with variable penetrance and expression.
(Choice B) The classic pedigree for autosomal recessive inheritance shows disease
development in 25% of offspring from two asymptomatic carrier parents. Typically
only one generation is affected.
(Choice D) Conditions passed by mitochondrial inheritance are transmitted only by
females. Affected males cannot pass the disorder to their offspring.
(Choice E) By definition, sporadic cases do not follow an inheritance pattern.
Sporadic cases occur haphazardly among unrelated members of a population
overtime. For example there maybe a spontaneous gene mutation that causes a
sporadic disorder in an individual with no family history of the disease.
Educational Objective:
A pedigree involving a female index case (proband), affected father, and affected
paternal grandmother most likely corresponds to autosomal dominant or X-linked
dominant inheritance. Androgenetic alopecia is the most common cause of hair loss
in both males and females and is generally inherited as an autosomal dominant or
polygenic disorder.

198

USMLE WORLD STEP 1

GENETICS

Q NO 32: A 14-year-old Caucasian male is brought to your office by his mother. She
is concerned because although tall her son looks much younger than his peers and
shows no signs of masculinity. On physical examination, the boy has poorlydeveloped secondary sexual characteristics. He is unable to distinguish smells, but
has good visual acuity. Which of the following pathways is most likely defective in
this patient?

A.
B.
C.
D.
E.

A
B
C
D
E

Explanation:
The male child in the vignette has delayed puberty plus anosmia, consistent with a
diagnosis of Kallmanns syndrome. Kallmanns syndrome results from a failure of
GnRH-secreting neurons to migrate from their origin in the olfactory placode
(situated outside the central nervous system) to their normal anatomic location in
the hypothalamus. Most often1 the cause is a mutation in either the KAL-1 gene or
the fibroblast growth factor receptor-i gene, which code for proteins required in this
migration.
Patients with Kallmanns syndrome classically have central hypogonadism and
anosmia, though there may be other midline defects as well (e.g. cleft lip or cleft
palate). Most often theses patients present with delayed puberty. On physical exam,
the testes are often just 1-2 ml in volume. There is usually some pubic hair because
adrenarche occurs normally.
In Caucasians delayed puberty is defined as the absence or incomplete
development of secondary sexual characteristics by age 14 in males and by age 12
in females. Testicular enlargement is the first sign of puberty in males and breast
enlargement is the first sign in females. The initiation of puberty and the

199

USMLE WORLD STEP 1

GENETICS

maintenance of secondary sexual characteristics and fertility require the coordinated


efforts of the hypothalamus1 pituitary and gonads. Delayed puberty can result from
a derangement anywhere in this axis.
(Choices B, C, D, and E) These pathways represent normal feedback loops between
the testes and the hypothalamus and pituitary. Pituitary FSH stimulates proliferation
of the seminiferous tubules and spermatogenesis. The seminiferous tubules produce
inhibin, which feeds back to inhibit pituitary ISH secretion. LH from the pituitary
stimulates the interstitial Leydig cells to produce testosterone1 which also
participates in feedback inhibition.
Educational Objective:
In Kallmanns syndrome, there is an absence of GnRH secretory neurons from the
hypothalamus due to defective migration from the olfactory placode. These patients
have central hypogonadism and anosmia.

200

USMLE WORLD STEP 1

GENETICS

Q NO 33: A 12-year-old Caucasian male with moderate mental retardation and a


dysmorphic face is found to have 226 CCC trinucleotide repeats in a gene located
on the X chromosome. Which of the following is the most likely cause of this
patients condition?
A. Exon deletion
B. Chromosomal breaks
C. Gene methylation
D. Impaired intron splicing
E. Mismatch repair detect
Explanation:
This patient has fragile X syndrome, the second most common cause of inherited
mental retardation in males after Down syndrome. Individuals with fragile X
syndrome often have facial dysmorphism characterized by a long thin face. a
prominent forehead and jaw large protruding ears, and tooth crowding.
Macroorchidism (large testes) is typical for affected males and typically occurs
during puberty.
The genetic defect responsible for fragile X syndrome is found on the long arm of the
X chromosome in the area called Familial Mental Retardation gene-i (EMP1).
Normally this gene has between 5 and 55 CGG trinucleotide repeats. Individuals with
an increased number of repeats that remains less than 200 are said to have a
permutation. Patients with the permutation are typically phenotypically normal. Full
mutation occurs when there are more than 200 CGG repeats on the FMR1 gene. The
increased number of trinucleotide repeats causes hyper methylation (Choice C) of
cysteine bases in the FMR1 gene leading to gene inactivation. Fragile X syndrome is
so named because when lymphocytes from affected individuals are cultured in a
folate- and thymidine-depleted medium and the karyotype analyzed this region of
the X-chromosome appears constricted and thin (or fragile).
(Choices A and D) Exons (Expressed codons) are coding regions of DNA, as
compared to introns which do not carry genetic information. Introns are removed
during intranuclear mRNA processing. There are no abnormalities of exon deletion in
Fragile X syndrome.
(Choice B) Xeroderma pigmentosum, ataxia-telangiectasia, Fanconis anemia, and
Blooms syndrome are diseases caused by defects in DNA repair enzymes.
(Choice E) Defects in DNA mismatch repair genes cause Lynch syndrome, a condition
characterized by an increased risk of hereditary non-polyposis colorectal cancer
(HNPCC) and extraintestinal malignancies.
Educational Objective:
The familial mental retardation gene 1 (FMR1) is located on the long arm of the X
chromosome. An increased number of CGG trinucleotide repeats leads to hyper
methetoin of cysteine bases and gene inactivation. This defect is the cause of fragile
X syndrome, which manifests with mental retardation1 facial dysmorphism, and
macroorchidism.

201

USMLE WORLD STEP 1

GENETICS

Q NO 34: A 15-year-old Caucasian male presents to your office with gait instability
and frequent falls. Physical examination reveals kyphoscoliosis, pes cavus, and
lower-extremity ataxia. Position and joint sensation are also impaired. If the
symptoms described are from an inherited disease this patient is most likely to die
of which of the following?
A. Liver failure
B. Cardiomyopathy
C. Renal disease
D. Intracranial bleeding
E. Aortic dissection
F. Brain tumor
Explanation:
Friedreich ataxia is an autosomal recessive condition. The posterior columns and the
spinocerebellar tracts of the spinal cord show degeneration; and loss of the larger
sensory cells of the dorsal root ganglia is also seen. Friedreich ataxia presents in
children 5-15 years old as gait ataxia (progressively slow and clumsy walking). A
wide-based gait with difficulty maintaining balance is characteristic.
The following symptoms are associated with Friedreich ataxia:
1. Progressive ataxia of all four limbs, caused by cerebellar dysfunction, is seen very
early. Degeneration of dorsal columns leads to loss of position and vibration
sensation.
2. Hypertrophic cardiomyopathy develops in more than 50% of patients. It leads to
cardiac arrhythmias and congestive heart failure (CHF).
3. Skeletal abnormalities include kyphoscoliosis, pes cavus, and hammertoes.
4. Diabetes mellitus develops in about 10% of the patients.
Complications from cardiomyopathy and bulbar dysfunction (unable to protect
airway) are the most common causes of death in patients with Friedreich ataxia.
The conditions mentioned in the other choices are not associated with Friedreich
ataxia.
(Choice A) Liver failure is associated with Wilson disease.
(Choice C) Kidney disease is associated with polycystic kidney disease, and presents
at later age.
(Choice D) Intracranial bleeding is associated with rupture of Berry aneurysm.
(Choice E) Aortic dissection is associated with Marfan syndrome.
(Choice F) Brain tumor (glioma) is associated with neurofibromatosis.
Educational Objective:
Friedreich ataxia is an autosomal recessive condition. The mutated gene on
chromosome 9 has an increased number of trinucleotide repeats. Friedreich ataxia is
often associated with hypertrophic cardiomyopathy, diabetes mellitus,
kyphoscoliosis, and foot deformities.

202

USMLE WORLD STEP 1

GENETICS

Q NO 35: A mother brings her 1-month-old son to the ER because he appears


floppy and has not been feeding well. The mother is a 48-year-old multipara
and the family recently emigrated from Eastern Europe. Physical examination
reveals mild jaundice an enlarged tongue, general hypotonia and an umbilical
hernia. Which of the following is the most likely cause of this patients
condition?
Hypothyroidism
B. Down syndrome
C. Botulism
D. Hirschsprung disease
E. Phenylketonuria

A.

Explanation:
The infant described above displays symptoms consistent with congenital
hypothyroidism. Affected neonates appear lethargic, feed poorly, exhibit prolonged
jaundice, and demonstrate constipation, muscle hypotonia, and a hoarse cry.
Physical examination of the infant with congenital hypothyroidism reveals pale dry,
cool skin; myxedema (edema of skin and subcutaneous fat); and macroglossia
(large tongue). Coarse facial features and umbilical hernia are commonly present as
well. These infants also have an increased incidence of congenital heart defects such
as ASD and VSD.
T4 is essential for normal brain development and myelination during early life1 and
undiagnosed congenital hypothyroidism produces profound and irreversible mental
retardation. This complication is prevented by screening newborns for congenital
hypothyroidism. (By law, newborns are also screened for phenylketonuria and
galactosemia.)
(Choice B) Infants with Down syndrome have upslanting palpebral fissures, bilateral
epicanthal folds, a flat nasal bridge and a large tongue. Other common features are
congenital heart defects, duodenal atresia and Hirschsprung disease. Down
syndrome is also associated with an increased risk of leukemia.
(Choice C) Infant botulism affects children 2-6 months of age who are fed honey. It
presents with constipation and poor sucking followed by muscle paralysis. Infant
botulism tends to be less severe than adult botulism.
(Choice D) Hirschsprung disease results from the abnormal migration of neural crest
cells into the rectosigmoid colon leading to an absence of ganglion cells in the
affected wall. It presents with abdominal distention bilious emesis, and failure to
pass meconium.
(Choice E) Phenylketonuria occurs due to an inborn deficiency of phenylalanine
hydroxylase that leads to an inability to metabolize phenylalanine. It manifests with
developmental delay mental retardation, mousy body odor and fair skin coloring.
Educational Objective:
Congenital hypothyroidism presents soon after birth with hypotonia, poor feeding,
jaundice, macroglossia, constipation and umbilical hernia. It should be diagnosed as
early as possible to prevent the development of mental retardation.

203

USMLE WORLD STEP 1

GENETICS

Q NO 36: A 23-year-old man is being evaluated for myoclonic epilepsy of recent


onset. The episodes are short-lived and triggered by startle. Physical
examination reveals proximal muscle weakness. Gomori trichrome stain of a
muscle biopsy specimen shows muscle fibers with a blotchy red appearance. No
family history is available because the patient is adopted. What is the probability
that this patients one offspring will inherit the disease?
A. 100%
B. 75%
C. 50%
D. 25%
E. 0%
Explanation:
Blotchy red muscle fibers on Gomori trichrome stain are characteristic of the
mitochondrial myopathies. In these conditions, abnormal mitochondria accumulate
under the sarcolemma of muscle fibers. The fibers have an irregular shape and size
on cross section. For this reason the mitochondrial myopathies are also known as the
red ragged fiber diseases. Electron microscopy of the affected muscle reveals an
increased number of enlarged, abnormally shaped mitochondria.
The most important mitochondrial myopathies are myoclonic epilepsy with ragged
red fibers (MERRF, described in this vignette), Leber optic neuropathy (blindness),
and mitochondrial encephalopathy with stroke-like episodes and lactic acidosis
(MELAS). Each of these diseases is the result of a mitochondrial gene mutation.
The mitochondrial diseases are caused by mutations, deletions or duplications of
mtDNA. Because sperm mitochondria do not pass into the ovum during fertilization
only maternal mitochondria are transmitted to the fetus. Mitochondrial diseases
therefore have maternal inheritance. For this reason the male patient described in
this vignette will not transmit the disease to his progeny.
(Choice B) No pure single-gene inherited disease shows 75% transmission. There
may be 75% transmission for a disease with variable penetrance.
Individuals affected by a disease with autosomal dominant inheritance receive one
copy of the abnormal gene from one parent. The chance that an affected person
wills transmit the disease to his/her child is 50% (Choice C) if the parent is
heterozygous for the given disease. If the parent is homozygous, there is a 100%
chance (Choice A) of transmission.
(Choice D) Autosomal recessive diseases occur when the individual inherits two
copies of the mutated gene (one from each parent). If both parents are carriers1
there is a 25% chance that the child will be affected.
Educational Objective:
Red ragged muscle fibers are seen in mitochondrial diseases. Muscle fibers have
this appearance because abnormal mitochondria accumulate under the sarcolemma.
Mitochondrial diseases show maternal inheritance.

204

USMLE WORLD STEP 1

GENETICS

Q NO 37: A 23-year-old asymptomatic male participates in clinical research and is


found to be homozygous for the apolipoprotein E-4 allele. In the future, this
patient is most likely to suffer from which of the following?
A. Familial hypercholesterolemia
B. Hypertrophic cardiomyopathy
C. Diabetes mellitus, type 2
D. Peptic ulcer disease
E. Polycystic kidney disease
F. Alzheimer dementia
Explanation:
Alzheimer disease has a strong genetic predisposition. About 30% of patients have a
family history of the disease. According to age when symptoms begin1 Alzheimer
disease can be classified as early or late. The following three mutation sites have
been associated with early-on set familial Alzheimer disease (onset < 60 years old):
1. Amyloid precursor protein (APP) gene on chromosome 21
2. Presenilin 1 gene on chromosome 14
3. Presenilin 2 gene on chromosome 1
Both APP and presenilin gene mutations are thought to promote the production of A
3-amyloid.
Late-on set familial Alzheimer disease is associated with the E4 allele of
Apolipoprotein E. The exact mechanism of its influence is not known. It is thought
that the ApoE4 protein maybe involved in the formation of senile plaques.
(Choice A) Familial hypercholesterolemia is an autosomal dominant disorder. It is
associated with a defect in low density lipoprotein (LDL) receptors. This defect leads
to decreased hepatic LDL uptake and severe elevation in total cholesterol and LDL
levels.
(Choice B) Hypertrophic cardiomyopathy is a familial disorder inherited as an
autosomal dominant trait. The most common mutation is of the 3-myosin heavy
chain.
(Choice C) Genetic factors play an important role in diabetes mellitus type 2 (more
important than in type 1). Genetic predisposition to increased insulin resistance and
deranged insulin secretion by 13-cells are important precursors to disease.
(Choice D) Peptic ulcer disease is strongly associated with H. pylon infection, but not
ApoF4.
(Choice E) Polycystic kidney disease can be inherited as autosomal recessive (also
called infantile form) or dominant trait (adult form). Neither of these conditions is
associated with ApoF4.
Educational Objective:
Early-onset familial Alzheimer disease is associated with three gene mutations: APP
(chromosome 21), presenilin 1 and presenilin 2. Late-onset familial Alzheimer
disease is associated with apolipoprotein F4 genotype.

205

USMLE WORLD STEP 1

GENETICS

Q NO 38: A 5-year-old male suffers from recurrent diarrhea that does not respond
to short-term antibiotic therapy. Light microscopy of repeated duodenal aspirates
consistently reveals the following:

Which of the following may be impaired in this patient?


A. Eosinophil-mediated cytotoxicity
B. CD8+ T lymphocyte-mediated cytotoxicity
C. Bacterial killing by neutrophils
D. Membrane attack complex formation
E. IgA production
Explanation:
The above picture shows a Giardia lamblia trophozoite. Giardia is pear-shaped,
bilaterally symmetric organisms with four pairs of flagella and two nuclei. Secretory
IgA helps prevent and clear infection by binding to trophozoites and impairing their
adherence to the upper small bowel mucosa. Patients deficient in IgA antibodies
against specific Giardia antigens are prone to chronic giardiasis. Children with Xlinked agammaglobulinemia and patients with common variable immune deficiency
have a documented predisposition to giardiasis. These patients also tend to have a
prolonged disease course with diarrhea malabsorption villus flattening crypt
hypertrophy, and dense submucosal mononuclear cell infiltrates on small bowel
biopsy.
(Choice A) Eosinophil-mediated c4otoxicitsI is not involved in the defense against
Giardia. Rather, this mechanism is important in combating helminthic infections.
Eosinophils damage helminth larvae via an antibody dependent cellular cytotoxicity
(ADCC) mechanism possibly involving major basic protein.
Educational Objective:
Giardia Iamblia causes injury to the duodenal and jejunal mucosa by adhering to the
intestinal brush border and releasing molecules that induce a mucosal inflammatory
response. Secretory IgA, which impairs adherence, is the major component of
adaptive immunity against G. lamblia infection. Severe IgA deficiency predisposes
patients to chronic giardiasis.

206

USMLE WORLD STEP 1


Q NO 39: A 14-year-old male is diagnosed with mild mental retardation.
Cytogenetic studies show a small gap near the tip of the long arm of the
X chromosome. Physical examination of this patient is likely to show:
stature, broad chest, and amenorrhea
B. Tall stature, gynecomastia, infertility
C. Macroorchidism, large jaw and ears
D. Arachnodactyly, scoliosis, lens dislocation
E. Short broad hands, transverse palmar crease

GENETICS

A. Short

Explanation:
Fragile X syndrome is the second most common cause of congenital mental
retardation after Down syndrome. It is inherited in an X-linked fashion. The defect is
an increased number of trinucleotide repeats (CGG) in the familial mental
retardation gene (FMR-1) located on the long arm of the X-chromosome. When the
cells of affected individuals are cultured in folate-deficient medium, the area of
increased repeats does not stain and appears broken hence the name fragile X.
Patients with fragile X syndrome display the following features:
1. Body habitus: Macrosomia with increased head circumference may be present at
birth. Older patients have dysmorphic facial features including large jaw, large
protruding ears, long thin face and prominent forehead. Postpubertal males
invariably have macroorchidism (enlarged testes).
2. Cognitive impairment: Becomes evident alter the 1st year of life. Patients
demonstrate mild-to-moderate mental retardation, severe language delay and
behavioral abnormalities (such as aggressiveness). Autistic features are more
common in children with fragile X syndrome than in the general population.
(Choice A) Short stature, broad (shield) chest, webbed neck and low hair line are
signs of Turners syndrome (45 XO). Affected females have primary amenorrhea due
to lack of estrogen secretion by underdeveloped (streak) ovaries.
(Choice B) Klinefelter syndrome (47 XXY) affects only males. Patients have tall
stature1 gynecomastia and small, firm testes. Decreased testosterone secretion by
fibrotic testes causes oligospermia and infertility.
(Choice D) In Marfan syndrome there is often arachnodactyly, scoliosis, and aortic
root dilation. Other associated abnormalities include lens dislocation (ectopia lentis)
and mitral valve prolapse.
(Choice E) Short broad fingers and transverse palmar crease are seen in Down
syndrome.
Educational Objective:
Fragile X syndrome is a common cause of inherited mental retardation. The disorder
is X-linked and affects males. Patients have mental retardation, dysmorphic facial
features (large jaw large protruding ears) and macroorchidism.

207

USMLE WORLD STEP 1

GENETICS

Q NO 40: A 32-year-old male with Down syndrome experiences significant cognitive


decline, making him totally dependant on a caregiver for his basic daily activities.
Cortical neurons in this patient are most likely to demonstrate which of the
following?
A. Pick bodies
B. Neurofibrillary tangles
C. Lewy bodies
D. Spongiform transformation
E. Negri bodies
F. Psammona bodies
Explanation:
Down syndrome (trisomy 21) is the most common chromosomal anomaly. Typical
symptoms are mental retardation and mongoloid appearance. Characteristic facial
features include flat facies, epicanthic folds and oblique palpebral fissures. Patients
with Down syndrome are unfortunately more prone to a number of medical
problems.
For instance, patients with Down syndrome have an increased risk of early-onset
Alzheimer disease. Beyond 40, these patients invariably develop senile plaques and
neuro fibrillary tangles. The association of Down syndrome and Alzheimer disease
maybe explained by the fact that the APP gene is located on chromosome 21,
Cleavage of APP (amyloid precursor protein) forms A -amyloid, which is the
substance that accumulates in brain tissue and vessel walls. Trisomy of 21 entails
that three copies of the APP gene are present. More APP is therefore produced in
Down syndrome, facilitating accelerated accumulation of amyloid and Alzheimer-like
changes.
Other conditions associated with Down syndrome include:
1. Acute leukemias: both AML and ALL.
2. Congenital heart disease: endocardial cushion defects, VSDI ASD.
3. Gastrointestinal defects: duodenal atresia and Hirschsprung disease.
(Choice A) Pick bodies are eosinophilic, intracytoplasmic inclusions found in the
cortical neurons in Pick disease. Down syndrome is not associated with increased
risk of Pick disease.
(Choice C) Lewy bodies are round, eosinophilic, intracytoplasmic inclusions that
contain neurofilaments. They are seen in the neurons of substantia nigra in
Parkinson disease.
(Choice D) Spongiform transformation is characteristic of Creutzfeldt-Jacob disease,
which manifests by rapidly progressive dementia and myoclonic jerks. This condition
may have an infectious etiology and is not associated with Down syndrome.
(Choice E) Negri bodies are found in patients with rabies infection. They are round,
eosinophilic inclusions seen in the cytoplasm of pyramidal neurons of the
hippocampus and cerebellar Purkinje cells. Negri bodies contain the rabies virus.
(Choice F) Psammoma bodies are laminar calcifications found in a number of tumors
(meningiomas, papillary thyroid carcinomas, and many others).
Educational Objective:
A -amyloid plays an important role in the development of Alzheimer disease. Its
precursor protein (APP) is coded by a gene located on chromosome 21. Patients with
trisomy 21 (Down syndrome) are likely to develop Alzheimer disease after age 40.

208

USMLE WORLD STEP 1

GENETICS

Q NO 41: A 12-year-old Caucasian female is brought to your office complaining of


difficulty walking. Physical examination reveals femoral pulses that are delayed
relative to the brachial pulses, and pulsatile vessels along her ribs and axilla. This
patients condition is most likely associated with:
A. Kartagener syndrome
B. Tuberous sclerosis
C. DiGeorge syndrome
D. Friedreichs ataxia
E. Marfan syndrome
F. Down syndrome
G. Turners syndrome
Explanation:
Diminished femoral pulses compared to brachial pulses, symptoms of inadequate
perfusion of the lower extremities during ambulation, and enlarged intercostal
arteries in a child/young adult are characteristic of adult-type congenital coarctation
of the aorta. Turner syndrome (an XO sex chromosome disorder) is associated with
coarctation of the aorta in girls.
(Choice A) In Kartagener syndrome, cilia are immotile due to a microtubular dynein
arm defect. Infertility, recurrent sinusitis, and bronchiectasis result. This syndrome is
associated with situs inversus, but is not known to be associated with aortic
coarctation.
(Choice B) Tuberous sclerosis is an autosomal dominant syndrome characterized by
cutaneous angiofibromas (adenoma sebaceum), seizures, and mental retardation.
Pathological lesions include central nervous system hamartomas and benign
neoplasms, renal and other visceral cysts, a variety of other hamartomas, and
cardiac rhabdomyomas.
(Choice C) DiGeorge syndrome is associated with at least 15% of cases of
interrupted aortic arch, a more extreme anomaly than aortic coarctation in which the
aortic arch is atretic ora segment of the arch is absent. In addition to poor lower
extremity pulses respiratory distress variable cyanosis and signs of congestive heart
failure develop during the first days of life.
(Choice D) Friedreichs ataxia is marked by spinocerebellar degeneration with
predominantly spinal ataxia. This can produce difficulty walking, but femoral pulses
are not affected. This autosomal recessive condition is associated with hypertrophic
cardiomyopathy.
(Choice E) Marfan syndrome is associated with cystic medial necrosis of the code,
which may result in dissecting aortic aneurysms and aortic valve incompetence.
Mitral valve prolapse may also result.
(Choice F) Down syndrome is associated with endocardial cushion defects that result
in ostium primum atrial septal defects (ASD) and regurgitant atrioventricular valves.
Educational Objective:
Diminished femoral pulses compared to brachial pulses, symptoms of inadequate
perfusion of the lower extremities during ambulation, and enlarged intercostal
arteries in a child/young adult are typical of adult-type congenital coarctation of the
aorta. Turners syndrome is associated with coarctation of the aorta in girls.
A variety of other autosomal and sex chromosomal inherited disorders are
associated with cardiovascular developmental defects and/or pathology. The major
associations are as follows:
Down syndrome: endocardial cushion defects (ostium primum ASDI regurgitant
atrioventricular valves)

209

USMLE WORLD STEP 1

DiGeorge syndrome: tetralogy of Fallot and interrupted aortic arch


Friedreichs ataxia: hypertrophic cardiomyopathy
Marfan syndrome: cystic medial necrosis of the aorta
Tuberous sclerosis: valvular obstruction due to cardiac rhabdomyomas

210

GENETICS

USMLE WORLD STEP 1

GENETICS

Q NO 42: A patient is suspected of having an inherited disorder. Pedigree analysis


shows the following pattern (see the slide below).

This patient is most likely to be suffering from which of the following conditions?
A. Hemophilia B
B. Huntingtons disease
C. Classical galactosemia
D. Lesch-Nyhan syndrome
E. Leber hereditary optic neuropathy
Explanation:
Each of the affected individuals on this pedigree has inherited the disorder from
asymptomatic parents, consistent with an autosomal recessive inheritance pattern.
Classical galactosemia is an autosomal recessive disease. Patients with galactosemia
are homozygous for a defective galactose-1-phosphate uridyltransferase gene. In
general, most enzyme deficiency conditions follow an autosomal recessive
inheritance pattern, whereas diseases due to defective non-catalytic proteins tend to
follow an autosomal dominant pattern.
(Choice A) Hemophilia B is an X-linked recessive Factor IX deficiency that affects
males.
(Choice B) Huntingtons disease is inherited in an autosomal dominant fashion.
Affected individuals generally have one affected parent. In this condition, there is
degeneration of the striatum (mainly the caudate nucleus and putamen).
(Choice D) Lesch-Nyhan syndrome is an X-linked recessive deficiency of
hypoxanthine phosphoribosyltransferase, the enzyme that promotes conversion of
hypoxanthine to IMP and guanine to GMP (purine salvage).
(Choice E) Leber hereditary optic neuropathy follows a mitochondrial inheritance
pattern.
Educational Objective:
On average autosomal recessive conditions affect 25% of offspring of asymptomatic
heterozygous carrier parents. Classical galactosemia is an autosomal recessive
disease.

211

USMLE WORLD STEP 1

GENETICS

Q NO 43: An infant born to a 35-year-old female has a flat nasal bridge and a small
mouth. Karyotype analysis on the infant reveals the following:

The patient is most likely to suffer from which of the following conditions?
A. Immotile cilia
B. Macroorchidism
C. Rickets
D. Red blood cell sickling
E. Acute lymphoblastic leukemia
F. Chronic myelogenous leukemia
Explanation:
This karyotype shows trisomy 21 (47XX, +21) indicating a diagnosis of Down
syndrome, the most common cause of congenital mental retardation. In most cases,
Down syndrome results from meiotic nondisjunction in the ovum; the parents are
generally genetically normal. Affected individuals may exhibit a simian crease,
abundant neck skin, prominent epicanthal folds, a flat facial profile, mental
retardation, a cleft palate, congenital heart disease, intestinal atresia, hypotonia,
and/or a gap between the first and second toes. Individuals with Down syndrome
have a 20-40 fold increased risk of developing acute lymphoblastic leukemia (ALL).
(Choice A) Immotile cilia are found in Kartagener syndrome, a condition caused in
most cases by an autosomal recessive mutation in the gene coding for the
microtubule associated protein, dynein. Male infertility, recurrent sinusitis, and
bronchiectasis result. Situs inversus may also be present.

212

USMLE WORLD STEP 1

GENETICS

(Choice B) Macroorchidism (large testes) typically develops in pubertal males with


fragile X syndrome, the second most common genetic cause of mental retardation.
On karyotype analysis of cells cultured in a folate-deficient medium, this X-linked
disorder shows a discontinuity of staining on the long arm of the X chromosome.
(Choice C) Rickets is not associated with Down syndrome.
(Choice D) Red blood cell sickling occurs in sickle cell anemia. The sickle cell
mutation is localized to individual - globin genes and does not result from trisomy
21.
(Choice F) Chronic myelogenous leukemia (CML) is associated with the Philadelphia
chromosome reciprocal translocation between the long arms of chromosomes 9 and
22. This translocation fuses the BCR gene of chromosome 22 to the ABL gene of
chromosome 9. Karyotype analysis might show elongation of the long arm of one
chromosome 9.
Educational Objective:
Trisomy 21 (47, XX, +21) is detectable by cytogenetic karyotype analysis and is the
most common cause of congenital mental retardation. Patients with Down syndrome
are 20-40 times more likely to develop ALL than the general population.

213

USMLE WORLD STEP 1

GENETICS

Q NO 44: A 14-year-old male experiences severe prolonged bleeding after having a


tooth extracted. He also has a history of multiple episodes of painful joint swelling
following minor trauma. Evaluation reveals that he has an inherited disorder. Now
his older sister who does not have this condition, asks about the risk that her own
children will be affected. The best probable estimate is:
A. Near 0
B. 1/2
C. 1/4
D. 1/8
E. 1/16
F. 1/32
Explanation:
The male patient described above has a history of excessive bleeding and
hemarthroses, suggesting a diagnosis of hemophilia A or B, both X-linked recessive
coagulation factor deficiencies. The probability that his sister will give birth to an
affected child can be calculated as follows:
Probability (p1) that sister is a carrier = 0.5
P2 that sister will have a male child = 0.5
P3 that offspring of female carrier will inherit the X chromosome with the
hemophilia gene = 0.5
The probability that the sister will have an affected son is thus the probability that all
three of the above events take place. i.e. the product of their individual probabilities,
p1 x p2 x p3 = 1/2 x 1/2 x 1/2 = 1/e.
Educational Objective:
The probability that a female sibling of a male affected by an X-linked recessive
disease will give birth to an affected child is 1/ e.

214

USMLE WORLD STEP 1

GENETICS

Q NO 45: A 16-year-old Caucasian female presents to your office with amenorrhea.


She has never had menses. She is 175 cm tall, weighs 65 kg (143 lb) and has fully
developed secondary sexual characteristics. Pelvic examination reveals a shortened
vaginal canal with a rudimentary uterus. Which of the following is the most likely
diagnosis in this patient?
A. Klinefelter syndrome
B. Turner syndrome
C. 21-hydroxqlase deficiency
D. 47 XXX karyotype
E. Marfan syndrome
F. Mullerian agenesis
G. Kallmann syndrome
Explanation:
In the sexually undifferentiated embryo (<6 weeks of gestation)1 two pairs of
genital ducts are present: the paramesonephric (Mullerian) ducts and the
mesonephric (Wolffian) ducts.
In the female fetus, the Mullerian ducts give rise to fallopian tubes, uterus1 cervix,
and upper vagina, while the Wolffian ducts degenerate from the lack of hormonal
stimulation. Abnormal transformation of the Mullerian ducts into the female genitalia
results in a wide range of defects that include (but are not limited to) the following:
hypoplasia or agenesis of the vagina and uterus; duplication of the vagina1 cervix
and uterus; and unicornuate, bicornuate and septate uterus.
Because the patient presented here has fully developed secondary sexual
characteristics, normal levels of estrogens and gonadotropins are to be expected.
When menses are absent in the context of normally functioning ovaries and a
normally functioning anterior pituitary gland, the condition is considered to be
eugonadotropic amenorrhea. The two most common causes of primary
eugonadotropic amenorrhea are imperforate hymen or Mullerian duct anomalies. The
latter condition is the more likely cause of this patients uterine hypoplasia.
(Choice A) Klinefelter syndrome (karyotype 47 XXY) affects males and is
characterized by tall stature, poorly developed secondary sexual characteristics,
atrophic testes and infertility.
(Choice B) Turner syndrome (karyotype 45XO) is a common cause of primary
amenorrhea. Affected individuals have short stature, webbed neck, shielded chest
and streaked (fibrotic) ovaries. They do not develop secondary sexual
characteristics.
(Choice C) 21-hydroxylase deficiency varies in severity and may manifest in
neonates or later in life (even in young adults). Adult-onset 21-hydroxylase
deficiency may present as amenorrhea, hirsutism and virilization. Hyponatremia and
hyperkalemia are characteristic findings.
(Choice D) The 47 XXX karyotype is not associated with amenorrhea, though some
affected individuals may have slightly decreased 10 scores.
(Choice E) Marfan syndrome is not associated with amenorrhea.
(Choice G) Kallmann syndrome occurs due to diminished synthesis of gonadotropic
hormones by the anterior pituitary gland. The condition presents with primary
amenorrhea, absent secondary sexual characteristics, and an olfactory sensory
defect.

215

USMLE WORLD STEP 1

GENETICS

Educational Objective:
Primary amenorrhea in a patient with fully developed secondary sexual
characteristics suggests the presence of an anatomic defect in the genital tract, such
as imperforate hymen or Mllerian duct abnormalities.

216

USMLE WORLD STEP 1

GENETICS

Q NO 46: A 23-year-old Caucasian male with mild mental retardation has large
ears, a long face, a prominent mandible, and large testes. His hand joints are
hyperextensible on physical examination. Which of the following is the most
likely diagnosis in this patient?
Klinefelter syndrome
B. Turner syndrome
C. 47 XYY karyotype
D. 47 XXX karyotype
E. Testicular feminization syndrome
F. Mllerian agenesis
G. Marfan syndrome
H. Fragile X syndrome

A.

Explanation:
This patient displays the clinical features of fragile X syndrome, which is the most
common cause of inherited mental retardation and the second leading overall cause
of congenital mental retardation after Down syndrome. A male that inherits one
abnormal X chromosome will always develop the disease, as he has only one X
chromosome. A female with one abnormal X chromosome generally has a milder
clinical presentation and more normal cognitive development because she inherits
two X chromosomes, with one of the two randomly inactivated in every cell. Overall
the prognosis of females with fragile X is highly variable and cannot be accurately
predicted.
Males with fragile X syndrome typically demonstrates several of the following signs:
1. Mild-to-moderate mental retardation (10 of 40-85), speech and language delay,
autistic behavior and ADHD
2. Long face, prominent jaws1 large ears, and cleft palate
3. Macroorchidism
4. Mitral valve prolapse
5. Short height, joint laxity, scoliosis, pes cavus, double-jointed thumbs, and a single
palmar crease
(Choice A) Klinefelter syndrome (41)0(Y) may be associated with mild mental
retardation or normal intelligence. The typical patient is a tall male with
gynecomastia, small testes, and infertility.
(Choice B) Females with Turner syndrome (45XO) present with primary amenorrhea
and mild mental retardation, though normal cognitive function is also possible. The
typical patient has short stature, webbed neck, shielded chest and ovarian
dysgenesis.
(Choice C) The 47XYY karyotype is characterized by tall stature, severe acne, and
mild delays in both motor and language development. Early research suggested that
these individuals were prone to heightened aggression as well, but more recent
studies suggest the increase in criminal activity among the 41 XYY populations is
actually secondary to lower mean IQs and associated poor judgment.
(Choice D) The 47XXX karyotype is typically clinically silent. Some affected
individuals may have slightly decreased IQ scores.
(Choice E) Testicular feminization syndrome (complete androgen insensitivity) is
characterized by normal external female genitalia but entirely absent wolffian and
mullerian structures. Mental retardation, facial deformities, and macroorchidism are
not seen.
(Choice F) Mllerian agenesis results in the congenital absence of the vagina and
variable uterine development. Mental retardation, facial deformities, and
macroorchidism are not seen.

217

USMLE WORLD STEP 1

GENETICS

(Choice G) Marfan syndrome is characterized by tall stature, arachnodactyly (long


and thin fingers), ectopia lentis (lens displacement), and dilatation of the proximal
aorta. Mental retardation facial deformities and macroorchidism are not seen.
Educational Objective:
Fragile X syndrome arises secondary to an increase in the number of trinucleotide
repeats within the FMR1 gene on the X chromosome. Typical clinical features of this
condition include mental retardation1 facial deformities, and macroorchidism.

218

USMLE WORLD STEP 1

GENETICS

Q NO 47: A 12-year-old Caucasian male with history of seizure disorder experiences


several stroke-like episodes with residual neurological deficit. He also suffers from
muscle weakness. Blood tests show increased serum lactate levels both postexercise and at rest. This patients condition in known to be maternally inherited.
This patients sister is also affected by the same disorder, but she displays very few
symptoms. Which of the following is the most likely explanation for the variability in
clinical presentation between the patient and his sister?
A. Genetic imprinting
B. Heteroplasmy
C. Anticipation
D. Low expression variability
E. Female sparing
Explanation:
Mitochondria have a small amount of their own DNA, called mitochondrial DNA
(mtDNA), which can have deletions and point mutations just like regular DNA.
Mitochondrial disorders (mutations in mtDNA) are unique in that they are exclusively
inherited from ones mother. Recall that the ovum is relatively large and has many
copies of mtDNA; whereas, the few copies of mtDNA present in sperm are lost
during fertilization. Mitochondrial diseases affect both male and female offspring with
equal frequency (100%), but there are variable degrees of severity. This variability
occurs because, during mitosis, mitochondria are randomly distributed between
daughter cells. As a result, some cells contain mitochondria with mostly damaged
mtDNA, while some contain mostly normal mitochondrial genomes. This mixture of
two types of genetic material is called Heteroplasmy and is responsible for the
clinical variability of mitochondrial diseases. The following mitochondrial syndromes
are important:
1. Leber hereditary optic neuropathy leads to bilateral vision loss.
2. Myoclonic epilepsy with ragged-red fibers: myoclonic seizures and myopathy
associated with exercise. Skeletal muscle biopsy shows irregularly shaped muscle
fibers (ragged red fibers).
3. Mitochondrial encephalomyopathy with lactic acidosis end stroke-like episodes
(MELAS). The clinical presentation of MELAS is described in this vignette.
(Choice A) Genetic imprinting is a selective inactivation of paternal or maternal
alleles. This phenomenon explains the differences in clinical presentation between
Prader-Willi and Angelman syndromes.
(Choice C) Anticipation refers to the increase in severity of genetic disorders in
subsequent generations. Anticipation is often seen diseases involving an increased
number of trinucleotide repeats, such as Huntington disease and Fragile X
syndrome. (Choice D) Variable expressivity refers to the differences in severity of
autosomal dominant disorders. For example, a patient with Marfan syndrome may
have only tall stature, while another patient with the same genetic defect will have
tall stature, aortic root dilation, and lens dislocation. Variable expressivity is not a
feature of mitochondrial diseases. (Choice E) Female sparing is not characteristic of
mitochondrial disorders. Both males and females are equally affected in
mitochondrial disorders.
Educational Objective: Mitochondrial diseases are characterized by exclusivelymaternal inheritance. The variable severity of these diseases is explained by the
random distribution of normal and mutated mitochondria between daughter cells
during mitosis; as a result, some cells may have completely healthy mitochondria,
while other cells contain mitochondria affected by genetic mutation (Heteroplasmy).
MELAS is a mitochondrial syndrome.

219

USMLE WORLD STEP 1

GENETICS

Q NO 48: When introduced individually1 two different mutant strains of adenovirus


do not cause cytopathic effects in a human cell culture. But when the cultured cells
are simultaneously exposed to both mutant strains, a new viral progeny strain is
produced that causes cellular enlargement and aggregation. Which of the following
most likely contributed to the formation of the progeny viral strain?
A. Reassortment
B. Recombination
C. Transformation
D. Phenotypic mixing
E. Interference
Explanation:
This scenario describes the exchange of genetic information between two virus
strains that have non-fragmented, double-stranded DNA genomes. Such an
exchange amounts to recombination, which may be defined as the exchange of
genes between two chromosomes via crossing over within homologous regions. The
resulting progeny will have traits not present simultaneously in either parent virus.
(Choice A) Reassortment refers to changes in genomic composition that occur when
host cells are co-infected with two segmented viruses that exchange whole genome
segments. This process can cause sudden alterations in the surface antigens of the
viral progeny, as observed with the highly mutagenic influenza virus. Because the
adenovirus genome contains non-segmented, double-stranded DNA, it does not
engage in reassortment.
(Choice C) Transformation is generally defined as the uptake of naked DNA by a
prokaryotic or eukaryotic cell. In virology this term may also be used to describe the
incorporation of viral DNA into a host cell chromosome. Transformation alters the
genetic composition of the host cell but typically causes no genomic change in
progeny virions.
(Choice D) Phenotypic mixing refers to co-infection of a host cell by two viral strains,
resulting in progeny virion that contains nucleocapsid proteins from one strain and
the genome of the other strain. In this case, since neither virion strain was
cytopathic (even though both strains could apparently insert their genomes into the
host cell line). Rearrangement of the capsid proteins around either unchanged
genome would not be expected to confer new cytopathic virulence to the progeny
virions.
(Choice E) Interference is the inhibition by one virus of the replication and/or release
of a second virus that is infecting the same cell. Simple interference would not result
in recombinant progeny virions.
Educational Objective:
Genome recombination between two defective viruses co-infecting the same host
cell can yield a cytopathic wild-type genome. Recombination is gene exchange that
occurs through the crossing over of two double-stranded DNA molecules.
Reassortment describes the mixing of genome segments in two or more segmented
viruses that infect the same host cell.

220

USMLE WORLD STEP 1

GENETICS

Q NO 49: An infant born to a 36-year-old Caucasian female demonstrates some


dysmorphic facial features and a holosystolic murmur at the left sternal border.
Karyotype analysis is consistent with trisomy 21. Which of the following
additional findings would be most expected in this infant?
Cleft palate
B. Polydactyly
C. Single palmar crease
D. Rocker-bottom feet
E. Macrocephaly
F. Macroorchidism

A.

Explanation:
Down syndrome is the most common autosomal trisomy identified in liveborn
infants. As many as 95% of Down syndrome cases arise due to chromosomal
nondisjunction during maternal meiosis (47 XX, +21) an abnormality that positively
correlates with increasing maternal age. Two of the more prominent and consistent
lectures of Down syndrome are mental retardation and facial dysmorphism. Almost
every organ and system, however, is affected. The characteristic Down syndrome
abnormalities are categorized by system below.

(Choice A) Cleft palate is associated with trisomy 13 (Patau syndrome).


(Choice B) Polydactylism associated with trisomy 13 (Patau syndrome).
(Choice D) Rocker-bottom feet are associated with both trisomy 13 (Patau
syndrome) and trisomy 18 (Edwards syndrome).
(Choice E) Macrocephaly is not a typical clinical feature of the more common
chromosomal syndromes.
(Choice F) Macroorchidism is associated with fragile X syndrome. Affected individuals
also have tall stature, large ears, long face, and mental retardation.
Educational Objective: Trisomy 21 (Down syndrome) is characterized by mental
retardation, facial dysmorphism, single palmar crease, endocardial cushion defects,
and duodenal atresia. Affected individuals have an increased risk of AML-M7 and ALL
in childhood and early Alzheimer disease in adulthood.

221

USMLE WORLD STEP 1

GENETICS

Q NO 50: A 25-year-old Caucasian male has a severe intolerance to certain


medications. On two occasions, his reactions to various medications have
necessitated hospital admission. His family pedigree with respect to this
condition is shown below.

This condition follows which inheritance pattern?


A. Autosomal dominant
B. Autosomal recessive
C. X-linked dominant
D. X-linked recessive
E. Mitochondrial
Explanation:
This pedigree shows that only males are affected by this drug intolerance.
Specifically, male offspring of unaffected parents are affected. There is no evidence
of male-to-male transmission. This pattern is most consistent with X linked recessive
inheritance from asymptomatic heterozygous (carrier) females. G6PD deficiency,
which causes an acute hemolytic anemia in reaction to oxidant drugs, follows an Xlinked recessive pattern of inheritance.
Autosomal dominant inheritance requires that at least one parent of the affected
individual express the trait as well. In autosomal recessive inheritance. 25% of
offspring of unaffected (heterozygous carrier) parents will bear the trait. In X linked
dominant inheritance, at least one parent (of either sex) is affected and all female
offspring of affected males have the disease. Finally, conditions with mitochondrial
inheritance are transmitted only by the mother, and all offspring of affected females
are likely to show signs of disease.
Educational Objective:
In X-linked recessive inheritance, male children of unaffected parents contract the
disease. There is no male-to-male transmission. G6PD deficiency, which causes an
acute hemolytic anemia in reaction to oxidant drugs, follows this pattern.

222

You might also like